Максимум и минимум функции без производной: Исследование функций без помощи производной | LAMPA

6)-8=6-8=-2.ymin​=log3​729−8=log3​(36)−8=6−8=−2. Наименьшее значение функции равно −2-2−2.

Содержание

Решение задач без нахождения производной

Здравствуйте! В этой статье речь пойдёт о задачах, которые можно решать без нахождения производной. В данной рубрике мы уже рассмотрели некоторые примеры с логарифмами, числом е, функции  с произведениями. Смысл заданий тот же –  требуется найти либо точку максимума (минимума) функции, либо определить максимальное (минимальное) значение функции. 

В чём суть и каков «стандартный» алгоритм решения — можно посмотреть в этой статье. Но не для всех заданий применение этого алгоритма будет рационально. Если следовать ему в представленных ниже примерах, то процесс решения будет «перегружен» вычислениями. А потеря времени на экзамене вам не нужна. Так какие же задания имеются ввиду?

В условии дана иррациональная, логарифмическая или показательная функция:

при чём под корнем, под знаком логарифма или в показателе находится квадратичная функция вида:

Рассмотрим подход без нахождения производной. Вы увидите, что такие задачи можно решать устно.

Что необходимо знать? Свойство параболы, напомним его:

Если а > 0, то её ветви направлены вверх.

Если а < 0, то её ветви направлены вниз.

Далее вспомним  координату  (абсциссу)  вершины параболы:

То есть, это точка экстремума квадратичной функции – в ней функция меняет своё поведение с возрастания на убывание или наоборот.

Следующий важный факт (ключевой для этих задач):

Если исходная функция монотонна (непрерывно возрастает или убывает), для нее указанная точка «х» также будет точкой экстремума.

Почему? Давайте рассмотрим отдельно функции подробнее.

Квадратичная функция в показателе степени (при чём n>1):

Смотрите! Представим, что ax2+bx+c=z. Можем записать:

Получается что значение z изменяется следующим образом.

Вариант когда a>0 (ветви параболы направлены вверх) – при х от минус бесконечности до –b/2a  z уменьшается, в точке –b/2a значение будет минимальным, далее при х от –b/2a  до бесконечности z увеличивается.

Это означает, что и сама функция у=nf(x) будет имет минимальное значение в точке х=–b/2a, так как при минимуме в показателе получится минимум в результате.

Вариант когда a<0 (ветви параболы направлены вниз) – при х от минус бесконечности до  –b/2a  z увеличивается, в точке –b/2a значение будет максимальным, далее при х от –b/2a  до бесконечности z уменьшается.

Это означает, что и сама функция у=nf(x) будет иметь максимальное значение в точке х=–b/2a, так как при максимуме в показателе получится максимум в результате.

Квадратичная функция под знаком логарифма (при чём n>1):

Представим, что ax2+bx+c=z. Можем записать:

Получается что значение z изменяется следующим образом:

Вариант когда a>0 (ветви параболы направлены вверх) –  при х от минус бесконечности до  –b/2a  z уменьшается, в точке –b/2a значение будет минимальным, далее при х от–b/2a  до бесконечности z увеличивается.

Это означает, что и сама функция lognz будет имет минимальное значение в точке х=–b/2a. Так как логарифмическая функция уменьшается при уменьшении аргумента (видно по графику).

Вариант когда a<0 (ветви параболы направлены вниз) – при х от минус бесконечности до  –b/2a  z увеличивается, в точке –b/2a значение будет максимальным, далее при х от –b/2a  до бесконечности z уменьшается.

Это означает, что и сама функция lognz будет имеет максимальное значение в точке х=–b/2a. Так как логарифмическая функция увеличивается при увеличении аргумента (видно по графику).

Квадратичная функция под знаком корня:

Представим, что ax2+bx+c=z. Можем записать:

Получается что:

При a>0 значение z минимально в точке х=–b/2a, а значит и сама функция будет иметь минимальное значение. *Корень из наименьшего значения в результате даст наименьшее число.

При a<0 значение z максимально в точке х=–b/2a, а значит и сама функция будет иметь максимальное значение.

Таким образом, сформулируем ключевое правило:

ВНИМАНИЕ! Конечно, если глубже уйти в тему, то возможны варианты когда сложная функция имеет отрицательный знак, когда логарифм находится в знаменателе дроби, когда основание логарифма или основание степени находится в пределах от 0 до 1. Разумеется,  важно понимать как ведёт себя данная в условии функция (возрастает или убывает). Но для решения типовых заданий экзамена указанного вывода вам будет вполне достаточно.

И конечно, не теряйте из виду область допустимых значений заданной функции:

— выражение стоящее под знаком корня, больше или равно нулю (число неотрицательное).

— выражение стоящее под знаком  логарифма, есть положительное число.

— выражение стоящее в знаменателе дроби не равно нулю.

В подобных задачах на нахождение наибольшего и наименьшего значения функции, я бы посоветовал находить область определения в любом случае (даже не смотря на то, что в представленных ниже примерах это ничего важного нам не даёт и не влияет на ответ).

Рассмотрим примеры:

*Контент (более шести решенных заданий) доступен только для зарегистрированных пользователей! Вкладка регистрации (входа) находится в ГЛАВНОМ МЕНЮ сайта. После прохождения регистрации войдите на сайт и обновите данную страницу.

В данной рубрике мы ещё рассмотрим задания с тригонометрическими функциями, не пропустите! Успеха вам!

С уважением, Александр

P.S: Буду благодарен Вам, если расскажите о сайте в социальных сетях.

Исследование функции с примерами решения

Содержание:

  1. Исследование функции без производной. Точки экстремума функции
  2. Пример с решением
  3. Исследование функции с помощью производной
  4. Целые рациональные функции

Исследование функции без производной. Точки экстремума функции

Точки экстремума — это точки, лежащие внутри области определения, в которых функция принимает самое большое (максимум) или самое малое (минимум) значение по сравнению со значениями в близких точках.

Точка называется точкой максимума (минимума) функции , если функция определена в самой этой точке и в некоторой окрестности точки выполняется неравенство . Обозначение: .

Значение функции в этих точках называют соответственно максимумами и минимумами (экстремумами) функции. Обозначение: .

Пусть функция определена на некотором множестве и имеет максимум в точке , , тогда имеют место следующие свойства:

  1. Для числа в точке функция имеет максимум, для числа — минимум.
  2. Для числа с в точке функция имеет максимум.
  3. Если или на множестве , то в точке функция имеет минимум.
  4. Если функция определена на множестве и имеет максимум в точке , то в точке функция имеет максимум.
  5. Если возрастающая функция определена на множестве , то в точке сложная функция имеет максимум.
  6. Если убывающая функция определена на множестве , то в точке сложная функция имеет минимум.

Аналогичные свойства имеют место для минимума функции на множестве , а также для ее наименьшего и наибольшего значений.

По этой ссылке вы найдёте полный курс лекций по высшей математике:

Напомним, что квадратичная функция при имеет минимум (наименьшее значение) , если ; имеет максимум (наибольшее значение) , если .

Квадратичная функция

при имеет минимум (наименьшее значение) , если ; имеет максимум (наибольшее значение) , если .

Пример с решением

Пример 1:

Найдите точку минимума функции .

Решение:

Квадратичная функция имеет в точке минимум . Так как функция возрастает на и , то данная сложная функция имеет минимум в точке .

Замечание. Выделяя полный квадрат в квадратном трехчлене, получим выражение , максимальное значение которого достигается при .

Замечание. Используя производную , , получаем, что при , при и при . Значит, — точка минимума.

Ответ: .

Возможно вам будут полезны данные страницы:

Исследование функции с помощью производной

Достаточное условие экстремума функции:

а) Если функция непрерывна в точке , a на интервале и на интервале , то точка является точкой минимума функции .

Упрощенная формулировка: если в точке производная меняет знак с минуса на плюс, то есть точка минимума.

б) Если функция непрерывна в точке , на интервале и на интервале , то точка является точкой максимума функции .

Упрощенная формулировка: если в точке производная меняет знак с плюса на минус, то есть точка максимума.

Внутренняя точка области определения функции, в которой производная равна нулю или не существует, называется критической.

Поведение функции при ее исследовании с помощью производной проиллюстрируем таблицами:

Второе достаточное условие экстремума функции. Пусть функция имеет в окрестности точки непрерывную вторую производную . Если и , то точка есть точка минимума функции . Если и , то точка есть точка максимума функции .

Отметим два способа решения задач на нахождение точек экстремума функции: применение первой производной или применение второй производной.

Целые рациональные функции

Пример 2:

Найдите точку максимума функции .

Решение:

1-й способ (использование первой производной).

1) Область определения функции .

2) Находим производную функции .

3) Решаем уравнение ; . Корни уравнения (и критические точки функции): и .

4) Расставляем знаки производной на каждом из интервалов , и . Например, . Ставим знак «-» на интервале . На остальных интервалах расставляем знаки, используя свойство знакочередования производной (см. рисунок). С помощью стрелок указываем характер монотонности функции .

При переходе через критическую точку производная меняет знак с плюса на минус. Значит, в силу непрерывности функции в этой точке, получаем, что — точка максимума.

При переходе через критическую точку производная меняет знак с минуса на плюс. Значит, — точка минимума.

2-й способ (использование второй производной). 1) .

2) ; .

3) ; . Корни уравнения (и критические точки функции): и .

4) Найдем вторую производную и вычислим значения при и при : ; ; . Так как , то — точка максимума данной функции. Так как , то — точка минимума данной функции.

Ответ: .

Наименьшее и наибольшее значения функции на отрезке

На рисунках ниже показано, где функция может достигать наименьшего и наибольшего значения. На левом рисунке наименьшее и наибольшее значения зафиксированы в точках локального минимума и максимума функции. На правом рисунке — на концах отрезка.

Если функция y = f(x) непрерывна на отрезке [ab], то она достигает на этом отрезке наименьшего и наибольшего значений. Это, как уже говорилось, может произойти либо в точках экстремума, либо на концах отрезка. Поэтому для нахождения наименьшего и наибольшего значений функции, непрерывной на отрезке [ab], нужно вычислить её значения во всех критических точках и на концах отрезка, а затем выбрать из них наименьшее и наибольшее. Кстати, будет полезным открыть в новом окне материал

Свойства и графики элементарных функций.

Пусть, например, требуется определить наибольшее значение функции f(x) на отрезке [ab]. Для этого следует найти все её критические точки, лежащие на [ab].

Критической точкой называется точка, в которой функция определена, а её производная либо равна нулю, либо не существует. Затем следует вычислить значения функции в критических точках. И, наконец, следует сравнить между собой по величине значения функции в критических точках и на концах отрезка (f(a) и f(b)). Наибольшее из этих чисел и будет наибольшим значением функции на отрезке [ab].

Аналогично решаются и задачи на нахождение

наименьших значений функции.

Для самопроверки при расчётах можно воспользоваться онлайн калькулятором производных.

Пример 1. Найти наименьшее и наибольшее значения функции на отрезке [-1, 2].

Решение. Находим производную данной функции . Приравняем производную нулю () и получим две критические точки: и . Для нахождения наименьшего и наибольшего значений функции на заданном отрезке достаточно вычислить её значения на концах отрезка и в точке , так как точка не принадлежит отрезку [-1, 2]. Эти значения функции — следующие: , , . Из этого следует, что наименьшее значение функции (на графике ниже обозначено красным), равное -7, достигается на правом конце отрезка — в точке , а наибольшее (тоже красное на графике), равно 9, — в критической точке .

Если функция непрерывна в некотором промежутке и этот промежуток не является отрезком (а является, например, интервалом; разница между интервалом и отрезком: граничные точки интервала не входят в интервал, а граничные точки отрезка входят в отрезок), то среди значений функции может и не быть наименьшего и наибольшего. Так, например, функция, изображённая на рисунке ниже, непрерывна на ]-∞, +∞[ и не имеет наибольшего значения.

Однако для любого промежутка (закрытого, открытого или бесконечного) справедливо следующее свойство непрерывных функций.

Если функция непрерывна в промежутке и имеет единственный экстремум, то он является наименьшим значением в случае минимума и наибольшим — в случае максимума.

Как наименьшее значение функции, так и её наибольшее значение, могут быть найдены не только в одной точке, принадлежащей заданного интервала, а, как, например, далее — в двух.

Нередки случаи, когда уравнение, полученное от приравнивания производной функции нулю, не имеет действительных решений. Тогда наименьшее и наибольшее значения функции можно найти только на концах отрезка. Таков следующий пример.

Неплохо было бы взять и случаи, когда производная функции вычисляется не одним махом, как в предыдущих примерах. Это мы сейчас и сделаем, решив пример, где требуется найти производную частного.

Для самопроверки при расчётах можно воспользоваться онлайн калькулятором производных.

Есть преподаватели, которые по теме нахождения наименьшего и наибольшего значений функции не дают студентам для решения примеры сложнее только что рассмотренных, то есть таких, в которых функция — многочлен либо дробь, числитель и знаменатель которой — многочлены. Но мы не ограничимся такими примерами, поскольку среди преподавателей бывают любители заставить студентов думать по полной (таблице производных). Поэтому в ход пойдут логарифм и тригонометрическая функция.

Для самопроверки при расчётах можно воспользоваться онлайн калькулятором производных.

В прикладных экстремальных задачах нахождение наименьшего (наибольшего) значений функции, как правило, сводится к нахождению минимума (максимума). Но больший практический интерес имеют не сами минимумы или максимумы, а те значения аргумента, при которых они достигаются. При решении прикладных задач возникает дополнительная трудность — составление функций, описывающих рассматриваемое явление или процесс.

Пример 10. Резервуар ёмкостью 4 , имеющий форму параллелепипеда с квадратным основанием и открытый сверху, нужно вылудить оловом. Каковы должны быть размеры резервуара, чтобы на его покрытие ушло наименьшее количество материала?

Решение. Пусть x — сторона основания, h — высота резервуара, S — площадь его поверхности без крышки, V — его объём. Площадь поверхности резервуара выражается формулой , т.е. является функцией двух переменных . Чтобы выразить S как функцию одной переменной, воспользуемся тем, что , откуда . Подставив найденное выражение h в формулу для S:

или

.

Исследуем эту функцию на экстремум. Она определена и дифференцируема всюду в ]0, +∞[, причём

.

Приравниваем производную нулю () и находим критическую точку . Кроме того, при производная не существует, но это значение не входит в область определения и поэтому не может быть точкой экстремума. Итак, — единственная критическая точка. Проверим её на наличие экстремума, используя второй достаточный признак. Найдём вторую производную . При вторая производная больше нуля (). Значит, при функция достигает минимума . Поскольку этот минимум — единственный экстремум данной функции, он и является её наименьшим значением. Итак, сторона основания резервуара должна быть равна 2 м, а его высота .

Для самопроверки при расчётах можно воспользоваться онлайн калькулятором производных.

Пример 11. Из пункта A, находящегося на линии железной дороги, в пункт С, отстоящий от неё на расстоянии l, должны переправляться грузы. Стоимость провоза весовой единицы на единицу расстояния по железной дороге равна , а по шоссе она равна . К какой точке М линии железной дороги следует провести шоссе, чтобы транспортировка груза из А в С была наиболее экономичной (участок АВ железной дороги предполагается прямолинейным)?

Пусть , , (см. рисунок ниже).

Тогда , , . Стоимость провоза p единиц груза по шоссе СМ составит , а по железной дороге МА она составит . Общая стоимость провоза груза по пути СМА выражается функцией

,

где .

Нужно найти наименьшее значение этой функции. Она дифференцируема при всех значениях x, причём

.

Приравняв производную нулю, получим иррациональное уравнение , решение которого даёт единственную критическую точку (так как точка не входит в область определения функции).

Взяв контрольные точки и слева и справа от критической точки, убедимся, что производная меняет знак с минуса на плюс. Следовательно, при стоимость провоза груза из А и С является наименьшей, если . Если же , т. е. , то шоссе должно пройти по прямой АС (см. рисунок ниже).

Весь блок «Производная»

Значения функции и точки максимума и минимума

Значения функции и точки максимума и минимума

Наибольшее значение функции 

Наменьшее значение функции 

Точки max 

Точки min

 


Как говорил крестный отец: «Ничего личного». Только производные!

Статью Как посчитать производные? надеюсь, ты изучил, без этого дальше будет проблематично.

12 задание по статистике считается достаточно трудным, а все потому, что ребята не прочитали эту статью (joke). В большинстве случаев виной всему невнимательность.

12 задание бывает двух видов:

  1. Найти точку максимума / минимума (просят найти значения «x»).
  2. Найти наибольшее / наименьшее значение функции (просят найти значения «y»).
Как же действовать в этих случаях?

Найти точку максимума / минимума

  1. Взять производную от предложенной функции.
  2. Приравнять ее к нулю.
  3. Найденный или найденные «х» и будут являться точками минимума или максимума.
  4. Определить с помощью метода интервалов знаки и выбрать, какая точка нужна в задании.

Задания с ЕГЭ: 

Найдите точку максимума функции 

  • Берем производную:

  • Приравняем ее к нулю:
  • Получили одно значение икса, для нахождения знаков подставим −20 слева от корня и 0 справа от корня в преобразованную производную (последняя строчка с преобразованием):

Все верно, сначала функция возрастает, затем убывает — это точка максимума!
Ответ: −15

Найдите точку минимума функции

  • Преобразуем и возьмем производную: 
  • Получается один корень «−2», однако не стоит забывать о «−3», она тоже будет влиять на изменение знака.

  • Отлично! Сначала функция убывает, затем возрасает — это точка минимума!
Ответ: −2

Найти наибольшее / наименьшее значение функции


  1. Взять производную от предложенной функции.
  2. Приравнять ее к нулю.
  3. Найденный «х» и будет являться точкой минимума или максимума.
  4. Определить с помощью метода интервала знаки и выбрать, какая точка нужна в задании.
  5. В таких заданиях всегда задается промежуток: иксы, найденные в пункте 3, должны входить в данный промежуток.
  6. Подставить в первоначальное уравнение полученную точку максимума или минимума, получаем наибольшее или наименьшее значение функции. 

Задания с ЕГЭ: 

Найдите наибольшее значение функции на отрезке [−4; −1] 

  • Преобразуем и возьмем производную: 
  • «3» не вдходит в промежуток [−4; −1]. Значит, остается проверить «−3» — это точка максимума?
  • Подходит, сначала функция возрастает, затем убывает — это точка максимума, и в ней будет наибольшее значение функции. Остается только подставить в первоначальную функцию:

Ответ: −6

Найдите наибольшее значение функции на отрезке [0; 1,5π]

  • Берем производную:
  • Находим, чему равняется sin(x):
  • Но такое невозможно! Sin(x)…
  • Получается, что уравнение не имеет решения, и в таких ситуациях нужно подставлять крайние значения промежутка в первоначальное уравнение:
  • Наибольшее значение функции равно «11» при точке максимума (на этом отрезке) «0».

Ответ: 11

Выводы:

  1. 70% ошибок заключается в том, что ребята не запоминают, что в ответ на наибольшее/наименьшее значение функции нужно написать «y», а на точку максимума/минимума написать «х».
  2. Нет решения у производной при нахождении значений функции? Не беда, подставляй крайние точки промежутка!
  3. Ответ всегда может быть записан в виде числа или десятичной дроби. Нет? Тогда перерешивай пример.
  4. В большинстве заданий будет получаться одна точка и наша лень проверять максимум или минимум будет оправдана. Получили одну точку — можно смело писать в ответ.
  5. А вот с поиском значения функции так поступать не стоит! Проверяйте, что это нужная точка, иначе крайние значения промежутка могут оказаться больше или меньше.

Будь в курсе новых статеек, видео и легкого математического юмора.

Точки экстремума функции, необходимые и достаточные условия экстремума

Содержание:

Определение

Точка $x_{0}$ называется точкой локального максимума функции $f(x)$, если существует такая окрестность этой точки, что для всех $x$ из этой окрестности выполняется неравенство: $f(x) \leq f\left(x_{0}\right)$.

Точка $x_{0}$ называется точкой локального минимума функции $f(x)$, если существует такая окрестность этой точки, что для всех $x$ из этой окрестности $f(x) \geq f\left(x_{0}\right)$.

Значение функции в точке максимума называется локальным максимумом, значение функции в точке минимума — локальным минимумом данной функции.{2}+1}=-1$.

Ответ. $y_{\min }=y(0)=-1$

Остались вопросы?

Здесь вы найдете ответы.

Что подразумевается под понятием «экстремум»?

Экстремум представляет собой значение функции на определенном интервале в момент достижения им минимального или максимального показания. Под понятием «экстремумы» или по-другому минимумы/максимумы подразумевается значение функции (у).

Точка экстремума – что это такое?

Если в определенной точке достигается экстремум или, иными словами, максимальное/минимальное значение функции на заданном интервале, то эта точка носит название точки экстремума. Из этого следует, что при достижении минимума, точка экстремума будет названа точкой минимума, и, наоборот, при достижении максимума эта точка будет называться точкой максимума. В случае, когда указываются точки экстремумов (или минимумов/максимумов) подразумеваются иксы, в которых достигаются минимальные или максимальные значения.

Что имеется в виду под понятием «точка минимума функции»?

Любая точка x₀ будет определена в качестве точки минимума функции y = f(x) при соблюдении условия о том, что имеется такая V, представляющая собой окрестность (x₀ — V; x₀+V) упомянутой ранее точки, из которой для каждого значения x x₀ действительно следующее неравенство:

f(x)>f(x₀).

Как описать точку минимума функции?

Под понятием «минимум функции» имеется в виду та точка на ней, в которой функция имеет значение, являющееся наименьшим среди всех значений, приобретаемых ею в любой из других соседних точек. Другими словами, это означает, что в случае, когда функция, достигнув определенной точки, прекращает падать, а, наоборот, наблюдается ее рост, то данная точка и представляет собой точку ее минимума.

Каким образом можно вычислить значение функции y=x⁴-4x³+6x²-4x, которого она достигает в точке своего минимума?

Для ответа на поставленный вопрос нужно отыскать точку минимума указанной функции, в которой ее значение перестает падать. Это можно сделать следующим образом:

y’ = 4x³ — 12x² + 12x – 4

Предположив, что минимальное значение данной функции равно 0, можно переписать равенство в следующем виде:

4x³ — 12x² + 12x — 4 = 0

Сократим данное уравнение на 4:

x³ — 3x² + 3x — 1 = 0

Получившееся равенство также может быть записано в следующем виде после перемены местами слагаемых:

(x³ — 1) + (-3x² + 3x) = 0

Распишем слагаемые в ином виде, чтобы избавиться от третьей степени:

(x — 1)(x² + x + 1) -3x(x — 1) = 0

Это же уравнение может выглядеть так:

(x -1)(x² + x + 1- 3x) = 0

Произведем сложение слагаемых х и -3х:

(x — 1) (x² -2x + 1) = 0

Теперь для упрощения можно переписать уравнение в таком виде:

(x — 1)(x-1)² = 0

Получившееся равенство:

(x — 1)³ = 0

В этом случае х = 1

-∞ 1 +∞

Знаками «+» и «-» обозначены значения производной.

После проведенных вычислений было установлено, что х = 1, что является точкой минимума функции:

у = 1⁴- 4*1³ + 6*1² — 4*1 = 1 — 4 +6 — 4 = -1

Какие расчеты нужно произвести, для того чтобы вычислить точку максимума для функции y = -x/x²+484?

Точкой максимума называется то значение х, достигнув которого, производная начинает менять свой знак с плюса на минус. Зная это, можно перейти к поиску точки максимума для функции, указанной в задании.

Для этого нужно начать с поиска производной, используя следующую формулу:

(U/V)’ = (U’V — UV’)/V²

Подставляем приведенные в задании значения и получаем:

y’ = (-(x² + 484) — 2x)/(x² + 484)² = (-x²-484 -2x)/(x² +484)²

Теперь следует приравнять производную к 0 и начать решать получившееся уравнение:

(-x²-484 -2x)/(x² +484)² = 0

Упростим уравнение и получим:

(-x²-484 -2x) = 0

(x² +484)² ≠ 0

-x²-484 -2x = 0

Избавимся от минусов в уравнении:

x² + 2x +484 = 0

D

В результате вычислений стало ясно, что корней нет. Это значит, что невозможно поставить их на числовой прямой, для того чтобы проверить знаки производной по соседству с этими точками. На основании этого можно сделать вывод о том, что указанная в задании функция не имеет точек экстремума.

Что представляет собой точка максимума функции?

Под точкой максимума функции понимается та точка, в которой она достигает значения, являющегося наибольшим среди тех значений, что достигаются ею в соседних точках. Это означает, что в точке, при пересечении которой функция прекращает расти, и наблюдается ее падение, и достигается ее максимум.

Имеется график производной функции. Каким образом можно вычислить точки ее максимума и минимума?

В случае, если имеется график производной функции, и при этом требуется определить ее экстремумы, то необходимо вычислить точки пересечения этого графика производной с осью Ох. По-другому они называются «нулями» производной. В случае, когда, пересекая конкретную точку, график производной восходит из области со знаком «-» в область со знаком «+», и в это время производная меняет свой знак на противоположный, функция также изменяется с убывания на рост. В этом случае данная точка, которая пересекается графиком производной, представляет собой точку минимума. Если же при пересечении графиком производной какой-либо точки он идет из положительной в отрицательную область, а функция из возрастания меняется на убывание, то речь идет о точке ее максимума.

Как можно вычислить экстремумы и точки экстремума функции y=4x⁴+2x²+1?

Для того чтобы найти ответ на поставленный вопрос, сначала нужно приравнять функцию к 0:

у = 0

Это же означает, что:

4X⁴ + 2X² + 1 = 0

Введем обозначения:

Х2 = А, при этом А больше 0.

С учетом введенных обозначений равенство будет иметь следующий вид:

4A² + 2A + 1 = 0

D = 4 — 4 = 0 ; √ D = 0

A = (- 2) : 4 = (- 0,5) (

Очевидно, что корней нет.

Ответ: х = 0, у = 1.

Дана функция y = x² -3x+2. Как можно вычислить экстремум этой функции?

Имеется функция y = x² -3x+2, которую также можно переписать в следующем виде:

у = -0,25+ (x-1,5)²

Отсюда следует, что:

miny = — 0,25 при условии, что х-1,5 = 0

Можно сделать вывод о том, что х = 1,5.

Запишем производную данной функции:

y ‘= (x² -3x+2)’ =2x -3

А затем приравняем ее к 0:

y ‘ = 0, значит:

2x -3 = 0.

Это позволяет сделать вывод о том, что:

x = 3/2.

Получается, что, если x

Если же x >3/2, то производная y’ > 0, и в этом случае функция возрастает.

x =3/2=1,5 – это единственная точка экстремума, которая является точкой минимума.

miny =(1,5)² -3*1,5+2 = -0,25.

Как раскрыть понятие «критическая точка функции»?

Критическая точка функции представляет собой ту точку, при пересечении с которой производная данной функции становится равной 0, либо она вовсе не существует.

Возможно ли привести доказательства того, что функция f(x) =2x — 3/x не может иметь критической точки?

Для начала нужно определить, что под критической точкой функции подразумевается та точка, при пересечении с которой производная приобретает нулевое значение, либо же эта производная просто не существует в этой точке, что означает, что функцию в данной точке невозможно дифференцировать.

Проверим, применимо ли это утверждение к упомянутой в задании функции:

f ‘(x) =(sin2x — 3x)’ = 2sin2x-3

Приравняем производную функции к 0:

f ‘(x) = 0, это значит, что 2sin2x-3 = 0.

Следовательно:

sin2x= 3 2 не имеет решения

Ответ: заданная функция не имеет критических точек и существует при любых х.

Каким способом можно определить критические точки функции y=|x|/1+x²?

Под критическими точками функции понимаются те точки, в которых ее производная равна 0 или вовсе не существует.

В задании дана функция:

y=|x|/(1+x²)

Предположим, что x

y=-x/(1+x²)

Запишем производную функции и приравняем ее к 0:

y`=(-1-x²+2x²)/(1+x²)²=(x²-1)/(1+x²)²=(x-1)(x+1)/(1+x²)²=0

х = 1 не соответствует условию, значит х = -1.

Теперь предположим, что x≥0.

Снова записываем производную имеющейся функции и приравниваем ее к 0:

y`=(1+x²-2x²)/(1+x²)²=(1-x²)/(1+x²)²=(1-x)(x+1)/(1+x²)²=0

х = — 1 не отвечает условию, значит х = 1.

Ответ: х = 1, х = -1.

Читать дальше: наибольшее и наименьшее значение функции.

Наибольшее и наименьшее значение функции теория. Исследование графика функции. Квадратичная функция записана через координаты вершины параболы

На практике довольно часто приходится использовать производную для того, чтобы вычислить самое большое и самое маленькое значение функции. Мы выполняем это действие тогда, когда выясняем, как минимизировать издержки, увеличить прибыль, рассчитать оптимальную нагрузку на производство и др., то есть в тех случаях, когда нужно определить оптимальное значение какого-либо параметра. Чтобы решить такие задачи верно, надо хорошо понимать, что такое наибольшее и наименьшее значение функции.

Обычно мы определяем эти значения в рамках некоторого интервала x , который может в свою очередь соответствовать всей области определения функции или ее части. Это может быть как отрезок [ a ; b ] , так и открытый интервал (a ; b) , (a ; b ] , [ a ; b) , бесконечный интервал (a ; b) , (a ; b ] , [ a ; b) либо бесконечный промежуток — ∞ ; a , (- ∞ ; a ] , [ a ; + ∞) , (- ∞ ; + ∞) .

В этом материале мы расскажем, как вычисляется наибольшее и наименьшее значение явно заданной функции с одной переменной y=f(x) y = f (x) .

Основные определения

Начнем, как всегда, с формулировки основных определений.

Определение 1

Наибольшее значение функции y = f (x) на некотором промежутке x – это значение m a x y = f (x 0) x ∈ X , которое при любом значении x x ∈ X , x ≠ x 0 делает справедливым неравенство f (x) ≤ f (x 0) .

Определение 2

Наименьшее значение функции y = f (x) на некотором промежутке x – это значение m i n x ∈ X y = f (x 0) , которое при любом значении x ∈ X , x ≠ x 0 делает справедливым неравенство f(X f (x) ≥ f (x 0) .

Данные определения являются достаточно очевидными. Еще проще можно сказать так: наибольшее значение функции – это ее самое большое значение на известном интервале при абсциссе x 0 , а наименьшее – это самое маленькое принимаемое значение на том же интервале при x 0 .

Определение 3

Стационарными точками называются такие значения аргумента функции, при которых ее производная обращается в 0 .

Зачем нам нужно знать, что такое стационарные точки? Для ответа на этот вопрос надо вспомнить теорему Ферма. Из нее следует, что стационарная точка – это такая точка, в которой находится экстремум дифференцируемой функции (т.е. ее локальный минимум или максимум). Следовательно, функция будет принимать наименьшее или наибольшее значение на некотором промежутке именно в одной из стационарных точек.

Еще функция может принимать наибольшее или наименьшее значение в тех точках, в которых сама функция является определенной, а ее первой производной не существует.

Первый вопрос, который возникает при изучении этой темы: во всех ли случаях мы может определить наибольшее или наименьшее значение функции на заданном отрезке? Нет, мы не можем этого сделать тогда, когда границы заданного промежутка будут совпадать с границами области определения, или если мы имеем дело с бесконечным интервалом. Бывает и так, что функция в заданном отрезке или на бесконечности будет принимать бесконечно малые или бесконечно большие значения. В этих случаях определить наибольшее и/или наименьшее значение не представляется возможным.

Более понятными эти моменты станут после изображения на графиках:

Первый рисунок показывает нам функцию, которая принимает наибольшее и наименьшее значения (m a x y и m i n y) в стационарных точках, расположенных на отрезке [ — 6 ; 6 ] .

Разберем подробно случай, указанный на втором графике. Изменим значение отрезка на [ 1 ; 6 ] и получим, что наибольшее значение функции будет достигаться в точке с абсциссой в правой границе интервала, а наименьшее – в стационарной точке.

На третьем рисунке абсциссы точек представляют собой граничные точки отрезка [ — 3 ; 2 ] . Они соответствуют наибольшему и наименьшему значению заданной функции.

Теперь посмотрим на четвертый рисунок. В нем функция принимает m a x y (наибольшее значение) и m i n y (наименьшее значение) в стационарных точках на открытом интервале (- 6 ; 6) .

Если мы возьмем интервал [ 1 ; 6) , то можно сказать, что наименьшее значение функции на нем будет достигнуто в стационарной точке. Наибольшее значение нам будет неизвестно. Функция могла бы принять наибольшее значение при x , равном 6 , если бы x = 6 принадлежала интервалу. Именно этот случай нарисован на графике 5 .

На графике 6 наименьшее значение данная функция приобретает в правой границе интервала (- 3 ; 2 ] , а о наибольшем значении мы не можем сделать определенных выводов.

На рисунке 7 мы видим, что функция будет иметь m a x y в стационарной точке, имеющей абсциссу, равную 1 . Наименьшего значения функция достигнет на границе интервала с правой стороны. На минус бесконечности значения функции будут асимптотически приближаться к y = 3 .

Если мы возьмем интервал x ∈ 2 ; + ∞ , то увидим, что заданная функция не будет принимать на нем ни наименьшего, ни наибольшего значения. Если x стремится к 2 , то значения функции будут стремиться к минус бесконечности, поскольку прямая x = 2 – это вертикальная асимптота. Если же абсцисса стремится к плюс бесконечности, то значения функции будут асимптотически приближаться к y = 3 . Именно этот случай изображен на рисунке 8 .

В этом пункте мы приведем последовательность действий, которую нужно выполнить для нахождения наибольшего или наименьшего значения функции на некотором отрезке.

  1. Для начала найдем область определения функции. Проверим, входит ли в нее заданный в условии отрезок.
  2. Теперь вычислим точки, содержащиеся в данном отрезке, в которых не существует первой производной. Чаще всего их можно встретить у функций, аргумент которых записан под знаком модуля, или у степенных функций, показатель которых является дробно рациональным числом.
  3. Далее выясним, какие стационарные точки попадут в заданный отрезок. Для этого надо вычислить производную функции, потом приравнять ее к 0 и решить получившееся в итоге уравнение, после чего выбрать подходящие корни. Если у нас не получится ни одной стационарной точки или они не будут попадать в заданный отрезок, то мы переходим к следующему шагу.
  4. Определим, какие значения будет принимать функция в заданных стационарных точках (если они есть), или в тех точках, в которых не существует первой производной (если они есть), либо же вычисляем значения для x = a и x = b .
  5. 5. У нас получился ряд значений функции, из которых теперь нужно выбрать самое больше и самое маленькое. Это и будут наибольшее и наименьшее значения функции, которые нам нужно найти.

Посмотрим, как правильно применить этот алгоритм при решении задач.

Пример 1

Условие: задана функция y = x 3 + 4 x 2 . Определите ее наибольшее и наименьшее значение на отрезках [ 1 ; 4 ] и [ — 4 ; — 1 ] .

Решение:

Начнем с нахождения области определения данной функции. В этом случае ей будет множество всех действительных чисел, кроме 0 . Иными словами, D (y) : x ∈ (- ∞ ; 0) ∪ 0 ; + ∞ . Оба отрезка, заданных в условии, будут находиться внутри области определения.

Теперь вычисляем производную функции согласно правилу дифференцирования дроби:

y » = x 3 + 4 x 2 » = x 3 + 4 » · x 2 — x 3 + 4 · x 2 » x 4 = = 3 x 2 · x 2 — (x 3 — 4) · 2 x x 4 = x 3 — 8 x 3

Мы узнали, что производная функции будет существовать во всех точках отрезков [ 1 ; 4 ] и [ — 4 ; — 1 ] .

Теперь нам надо определить стационарные точки функции. Сделаем это с помощью уравнения x 3 — 8 x 3 = 0 . У него есть только один действительный корень, равный 2 . Он будет стационарной точкой функции и попадет в первый отрезок [ 1 ; 4 ] .

Вычислим значения функции на концах первого отрезка и в данной точке, т.е. для x = 1 , x = 2 и x = 4:

y (1) = 1 3 + 4 1 2 = 5 y (2) = 2 3 + 4 2 2 = 3 y (4) = 4 3 + 4 4 2 = 4 1 4

Мы получили, что наибольшее значение функции m a x y x ∈ [ 1 ; 4 ] = y (2) = 3 будет достигнуто при x = 1 , а наименьшее m i n y x ∈ [ 1 ; 4 ] = y (2) = 3 – при x = 2 .

Второй отрезок не включает в себя ни одной стационарной точки, поэтому нам надо вычислить значения функции только на концах заданного отрезка:

y (- 1) = (- 1) 3 + 4 (- 1) 2 = 3

Значит, m a x y x ∈ [ — 4 ; — 1 ] = y (- 1) = 3 , m i n y x ∈ [ — 4 ; — 1 ] = y (- 4) = — 3 3 4 .

Ответ: Для отрезка [ 1 ; 4 ] — m a x y x ∈ [ 1 ; 4 ] = y (2) = 3 , m i n y x ∈ [ 1 ; 4 ] = y (2) = 3 , для отрезка [ — 4 ; — 1 ] — m a x y x ∈ [ — 4 ; — 1 ] = y (- 1) = 3 , m i n y x ∈ [ — 4 ; — 1 ] = y (- 4) = — 3 3 4 .

См. на рисунке:


Перед тем как изучить данный способ, советуем вам повторить, как правильно вычислять односторонний предел и предел на бесконечности, а также узнать основные методы их нахождения. Чтобы найти наибольшее и/или наименьшее значение функции на открытом или бесконечном интервале, выполняем последовательно следующие действия.

  1. Для начала нужно проверить, будет ли заданный интервал являться подмножеством области определения данной функции.
  2. Определим все точки, которые содержатся в нужном интервале и в которых не существует первой производной. Обычно они бывают у функций, где аргумент заключен в знаке модуля, и у степенных функций с дробно рациональным показателем. Если же эти точки отсутствуют, то можно переходить к следующему шагу.
  3. Теперь определим, какие стационарные точки попадут в заданный промежуток. Сначала приравняем производную к 0 , решим уравнение и подберем подходящие корни. Если у нас нет ни одной стационарной точки или они не попадают в заданный интервал, то сразу переходим к дальнейшим действиям. Их определяет вид интервала.
  • Если интервал имеет вид [ a ; b) , то нам надо вычислить значение функции в точке x = a и односторонний предел lim x → b — 0 f (x) .
  • Если интервал имеет вид (a ; b ] , то нам надо вычислить значение функции в точке x = b и односторонний предел lim x → a + 0 f (x) .
  • Если интервал имеет вид (a ; b) , то нам надо вычислить односторонние пределы lim x → b — 0 f (x) , lim x → a + 0 f (x) .
  • Если интервал имеет вид [ a ; + ∞) , то надо вычислить значение в точке x = a и предел на плюс бесконечности lim x → + ∞ f (x) .
  • Если интервал выглядит как (- ∞ ; b ] , вычисляем значение в точке x = b и предел на минус бесконечности lim x → — ∞ f (x) .
  • Если — ∞ ; b , то считаем односторонний предел lim x → b — 0 f (x) и предел на минус бесконечности lim x → — ∞ f (x)
  • Если же — ∞ ; + ∞ , то считаем пределы на минус и плюс бесконечности lim x → + ∞ f (x) , lim x → — ∞ f (x) .
  1. В конце нужно сделать вывод на основе полученных значений функции и пределов. Здесь возможно множество вариантов. Так, если односторонний предел равен минус бесконечности или плюс бесконечности, то сразу понятно, что о наименьшем и наибольшем значении функции сказать ничего нельзя. Ниже мы разберем один типичный пример. Подробные описания помогут вам понять, что к чему. При необходимости можно вернуться к рисункам 4 — 8 в первой части материала.
Пример 2

Условие: дана функция y = 3 e 1 x 2 + x — 6 — 4 . Вычислите ее наибольшее и наименьшее значение в интервалах — ∞ ; — 4 , — ∞ ; — 3 , (- 3 ; 1 ] , (- 3 ; 2) , [ 1 ; 2) , 2 ; + ∞ , [ 4 ; + ∞) .

Решение

Первым делом находим область определения функции. В знаменателе дроби стоит квадратный трехчлен, который не должен обращаться в 0:

x 2 + x — 6 = 0 D = 1 2 — 4 · 1 · (- 6) = 25 x 1 = — 1 — 5 2 = — 3 x 2 = — 1 + 5 2 = 2 ⇒ D (y) : x ∈ (- ∞ ; — 3) ∪ (- 3 ; 2) ∪ (2 ; + ∞)

Мы получили область определения функции, к которой принадлежат все указанные в условии интервалы.

Теперь выполним дифференцирование функции и получим:

y » = 3 e 1 x 2 + x — 6 — 4 » = 3 · e 1 x 2 + x — 6 » = 3 · e 1 x 2 + x — 6 · 1 x 2 + x — 6 » = = 3 · e 1 x 2 + x — 6 · 1 » · x 2 + x — 6 — 1 · x 2 + x — 6 » (x 2 + x — 6) 2 = — 3 · (2 x + 1) · e 1 x 2 + x — 6 x 2 + x — 6 2

Следовательно, производные функции существуют на всей области ее определения.

Перейдем к нахождению стационарных точек. Производная функции обращается в 0 при x = — 1 2 . Это стационарная точка, которая находится в интервалах (- 3 ; 1 ] и (- 3 ; 2) .

Вычислим значение функции при x = — 4 для промежутка (- ∞ ; — 4 ] , а также предел на минус бесконечности:

y (- 4) = 3 e 1 (- 4) 2 + (- 4) — 6 — 4 = 3 e 1 6 — 4 ≈ — 0 . 456 lim x → — ∞ 3 e 1 x 2 + x — 6 = 3 e 0 — 4 = — 1

Поскольку 3 e 1 6 — 4 > — 1 , значит, m a x y x ∈ (- ∞ ; — 4 ] = y (- 4) = 3 e 1 6 — 4 . Это не дает нам возможности однозначно определить наименьшее значение функции. Мы можем только сделать вывод, что внизу есть ограничение — 1 , поскольку именно к этому значению функция приближается асимптотически на минус бесконечности.

Особенностью второго интервала является то, что в нем нет ни одной стационарной точки и ни одной строгой границы. Следовательно, ни наибольшего, ни наименьшего значения функции мы вычислить не сможем. Определив предел на минус бесконечности и при стремлении аргумента к — 3 с левой стороны, мы получим только интервал значений:

lim x → — 3 — 0 3 e 1 x 2 + x — 6 — 4 = lim x → — 3 — 0 3 e 1 (x + 3) (x — 3) — 4 = 3 e 1 (- 3 — 0 + 3) (- 3 — 0 — 2) — 4 = = 3 e 1 (+ 0) — 4 = 3 e + ∞ — 4 = + ∞ lim x → — ∞ 3 e 1 x 2 + x — 6 — 4 = 3 e 0 — 4 = — 1

Значит, значения функции будут расположены в интервале — 1 ; + ∞

Чтобы найти наибольшее значение функции в третьем промежутке, определим ее значение в стационарной точке x = — 1 2 , если x = 1 . Также нам надо будет знать односторонний предел для того случая, когда аргумент стремится к — 3 с правой стороны:

y — 1 2 = 3 e 1 — 1 2 2 + — 1 2 — 6 — 4 = 3 e 4 25 — 4 ≈ — 1 . 444 y (1) = 3 e 1 1 2 + 1 — 6 — 4 ≈ — 1 . 644 lim x → — 3 + 0 3 e 1 x 2 + x — 6 — 4 = lim x → — 3 + 0 3 e 1 (x + 3) (x — 2) — 4 = 3 e 1 — 3 + 0 + 3 (- 3 + 0 — 2) — 4 = = 3 e 1 (- 0) — 4 = 3 e — ∞ — 4 = 3 · 0 — 4 = — 4

У нас получилось, что наибольшее значение функция примет в стационарной точке m a x y x ∈ (3 ; 1 ] = y — 1 2 = 3 e — 4 25 — 4 . Что касается наименьшего значения, то его мы не можем определить. Все, что нам известно, – это наличие ограничения снизу до — 4 .

Для интервала (- 3 ; 2) возьмем результаты предыдущего вычисления и еще раз подсчитаем, чему равен односторонний предел при стремлении к 2 с левой стороны:

y — 1 2 = 3 e 1 — 1 2 2 + — 1 2 — 6 — 4 = 3 e — 4 25 — 4 ≈ — 1 . 444 lim x → — 3 + 0 3 e 1 x 2 + x — 6 — 4 = — 4 lim x → 2 — 0 3 e 1 x 2 + x — 6 — 4 = lim x → — 3 + 0 3 e 1 (x + 3) (x — 2) — 4 = 3 e 1 (2 — 0 + 3) (2 — 0 — 2) — 4 = = 3 e 1 — 0 — 4 = 3 e — ∞ — 4 = 3 · 0 — 4 = — 4

Значит, m a x y x ∈ (- 3 ; 2) = y — 1 2 = 3 e — 4 25 — 4 , а наименьшее значение определить невозможно, и значения функции ограничены снизу числом — 4 .

Исходя из того, что у нас получилось в двух предыдущих вычислениях, мы можем утверждать, что на интервале [ 1 ; 2) наибольшее значение функция примет при x = 1 , а найти наименьшее невозможно.

На промежутке (2 ; + ∞) функция не достигнет ни наибольшего, ни наименьшего значения, т.е. она будет принимать значения из промежутка — 1 ; + ∞ .

lim x → 2 + 0 3 e 1 x 2 + x — 6 — 4 = lim x → — 3 + 0 3 e 1 (x + 3) (x — 2) — 4 = 3 e 1 (2 + 0 + 3) (2 + 0 — 2) — 4 = = 3 e 1 (+ 0) — 4 = 3 e + ∞ — 4 = + ∞ lim x → + ∞ 3 e 1 x 2 + x — 6 — 4 = 3 e 0 — 4 = — 1

Вычислив, чему будет равно значение функции при x = 4 , выясним, что m a x y x ∈ [ 4 ; + ∞) = y (4) = 3 e 1 14 — 4 , и заданная функция на плюс бесконечности будет асимптотически приближаться к прямой y = — 1 .

Сопоставим то, что у нас получилось в каждом вычислении, с графиком заданной функции. На рисунке асимптоты показаны пунктиром.

Это все, что мы хотели рассказать о нахождении наибольшего и наименьшего значения функции. Те последовательности действий, которые мы привели, помогут сделать необходимые вычисления максимально быстро и просто. Но помните, что зачастую бывает полезно сначала выяснить, на каких промежутках функция будет убывать, а на каких возрастать, после чего можно делать дальнейшие выводы. Так можно более точно определить наибольшее и наименьшее значение функции и обосновать полученные результаты.

Если вы заметили ошибку в тексте, пожалуйста, выделите её и нажмите Ctrl+Enter

Посмотрим, как исследовать функцию с помощью графика. Оказывается, глядя на график, можно узнать всё, что нас интересует, а именно:

  • область определения функции
  • область значений функции
  • нули функции
  • промежутки возрастания и убывания
  • точки максимума и минимума
  • наибольшее и наименьшее значение функции на отрезке.

Уточним терминологию:

Абсцисса — это координата точки по горизонтали.
Ордината — координата по вертикали.
Ось абсцисс — горизонтальная ось, чаще всего называемая ось .
Ось ординат — вертикальная ось, или ось .

Аргумент — независимая переменная, от которой зависят значения функции. Чаще всего обозначается .
Другими словами, мы сами выбираем , подставляем в формулу функции и получаем .

Область определения функции — множество тех (и только тех) значений аргумента , при которых функция существует.
Обозначается: или .

На нашем рисунке область определения функции — это отрезок . Именно на этом отрезке нарисован график функции. Только здесь данная функция существует.

Область значений функции — это множество значений, которые принимает переменная . На нашем рисунке это отрезок — от самого нижнего до самого верхнего значения .

Нули функции — точки, где значение функции равно нулю, то есть . На нашем рисунке это точки и .

Значения функции положительны там, где . На нашем рисунке это промежутки и .
Значения функции отрицательны там, где . У нас это промежуток (или интервал) от до .

Важнейшие понятия — возрастание и убывание функции на некотором множестве . В качестве множества можно взять отрезок , интервал , объединение промежутков или всю числовую прямую.

Функция возрастает

Иными словами, чем больше , тем больше , то есть график идет вправо и вверх.

Функция убывает на множестве , если для любых и , принадлежащих множеству , из неравенства следует неравенство .

Для убывающей функции большему значению соответствует меньшее значение . График идет вправо и вниз.

На нашем рисунке функция возрастает на промежутке и убывает на промежутках и .

Определим, что такое точки максимума и минимума функции .

Точка максимума — это внутренняя точка области определения, такая, что значение функции в ней больше, чем во всех достаточно близких к ней точках.
Другими словами, точка максимума — такая точка, значение функции в которой больше , чем в соседних. Это локальный «холмик» на графике.

На нашем рисунке — точка максимума.

Точка минимума — внутренняя точка области определения, такая, что значение функции в ней меньше, чем во всех достаточно близких к ней точках.
То есть точка минимума — такая, что значение функции в ней меньше, чем в соседних. На графике это локальная «ямка».

На нашем рисунке — точка минимума.

Точка — граничная. Она не является внутренней точкой области определения и потому не подходит под определение точки максимума. Ведь у нее нет соседей слева. Точно так же и на нашем графике не может быть точкой минимума.

Точки максимума и минимума вместе называются точками экстремума функции . В нашем случае это и .

А что делать, если нужно найти, например, минимум функции на отрезке ? В данном случае ответ: . Потому что минимум функции — это ее значение в точке минимума.

Аналогично, максимум нашей функции равен . Он достигается в точке .

Можно сказать, что экстремумы функции равны и .

Иногда в задачах требуется найти наибольшее и наименьшее значения функции на заданном отрезке. Они не обязательно совпадают с экстремумами.

В нашем случае наименьшее значение функции на отрезке равно и совпадает с минимумом функции. А вот наибольшее ее значение на этом отрезке равно . Оно достигается в левом конце отрезка.

В любом случае наибольшее и наименьшее значения непрерывной функции на отрезке достигаются либо в точках экстремума, либо на концах отрезка.

Иногда в задачах B15 попадаются «плохие» функции, для которых сложно найти производную. Раньше такое было лишь на пробниках, но сейчас эти задачи настолько распространены, что уже не могут быть игнорированы при подготовке к настоящему ЕГЭ.

В этом случае работают другие приемы, один из которых — монотонность .

Функция f (x ) называется монотонно возрастающей на отрезке , если для любых точек x 1 и x 2 этого отрезка выполняется следующее:

x 1 x 1 ) x 2 ).

Функция f (x ) называется монотонно убывающей на отрезке , если для любых точек x 1 и x 2 этого отрезка выполняется следующее:

x 1 x 1 ) > f (x 2 ).

Другими словами, для возрастающей функции чем больше x , тем больше f (x ). Для убывающей функции все наоборот: чем больше x , тем меньше f (x ).

Например, логарифм монотонно возрастает, если основание a > 1, и монотонно убывает, если 0 0.

f (x ) = log a x (a > 0; a ≠ 1; x > 0)

Арифметический квадратный (и не только квадратный) корень монотонно возрастает на всей области определения:

Показательная функция ведет себя аналогично логарифму: растет при a > 1 и убывает при 0 0:

f (x ) = a x (a > 0)

Наконец, степени с отрицательным показателем. Можно записывать их как дробь. Имеют точку разрыва, в которой монотонность нарушается.

Все эти функции никогда не встречаются в чистом виде. В них добавляют многочлены, дроби и прочий бред, из-за которого становится тяжело считать производную. Что при этом происходит — сейчас разберем.

Координаты вершины параболы

Чаще всего аргумент функции заменяется на квадратный трехчлен вида y = ax 2 + bx + c . Его график — стандартная парабола, в которой нас интересуют:

  1. Ветви параболы — могут уходить вверх (при a > 0) или вниз (a
  2. Вершина параболы — точка экстремума квадратичной функции, в которой эта функция принимает свое наименьшее (для a > 0) или наибольшее (a

Наибольший интерес представляет именно вершина параболы , абсцисса которой рассчитывается по формуле:

Итак, мы нашли точку экстремума квадратичной функции. Но если исходная функция монотонна, для нее точка x 0 тоже будет точкой экстремума. Таким образом, сформулируем ключевое правило:

Точки экстремума квадратного трехчлена и сложной функции, в которую он входит, совпадают. Поэтому можно искать x 0 для квадратного трехчлена, а на функцию — забить.

Из приведенных рассуждений остается непонятным, какую именно точку мы получаем: максимума или минимума. Однако задачи специально составляются так, что это не имеет значения. Судите сами:

  1. Отрезок в условии задачи отсутствует. Следовательно, вычислять f (a ) и f (b ) не требуется. Остается рассмотреть лишь точки экстремума;
  2. Но таких точек всего одна — это вершина параболы x 0 , координаты которой вычисляются буквально устно и без всяких производных.

Таким образом, решение задачи резко упрощается и сводится всего к двум шагам:

  1. Выписать уравнение параболы y = ax 2 + bx + c и найти ее вершину по формуле: x 0 = −b /2a ;
  2. Найти значение исходной функции в этой точке: f (x 0). Если никаких дополнительных условий нет, это и будет ответом.

На первый взгляд, этот алгоритм и его обоснование могут показаться сложными. Я намеренно не выкладываю «голую» схему решения, поскольку бездумное применение таких правил чревато ошибками.

Рассмотрим настоящие задачи из пробного ЕГЭ по математике — именно там данный прием встречается чаще всего. Заодно убедимся, что таким образом многие задачи B15 становятся почти устными.

Под корнем стоит квадратичная функция y = x 2 + 6x + 13. График этой функции − парабола ветвями вверх, поскольку коэффициент a = 1 > 0.

Вершина параболы:

x 0 = −b /(2a ) = −6/(2 · 1) = −6/2 = −3

Поскольку ветви параболы направлены вверх, в точке x 0 = −3 функция y = x 2 + 6x + 13 принимает наименьшее значение.

Корень монотонно возрастает, значит x 0 — точка минимума всей функции. Имеем:

Задача. Найдите наименьшее значение функции:

y = log 2 (x 2 + 2x + 9)

Под логарифмом снова квадратичная функция: y = x 2 + 2x + 9. График — парабола ветвями вверх, т.к. a = 1 > 0.

Вершина параболы:

x 0 = −b /(2a ) = −2/(2 · 1) = −2/2 = −1

Итак, в точке x 0 = −1 квадратичная функция принимает наименьшее значение. Но функция y = log 2 x — монотонная, поэтому:

y min = y (−1) = log 2 ((−1) 2 + 2 · (−1) + 9) = … = log 2 8 = 3

В показателе стоит квадратичная функция y = 1 − 4x − x 2 . Перепишем ее в нормальном виде: y = −x 2 − 4x + 1.

Очевидно, что график этой функции — парабола, ветви вниз (a = −1

x 0 = −b /(2a ) = −(−4)/(2 · (−1)) = 4/(−2) = −2

Исходная функция — показательная, она монотонна, поэтому наибольшее значение будет в найденной точке x 0 = −2:

Внимательный читатель наверняка заметит, что мы не выписывали область допустимых значений корня и логарифма. Но этого и не требовалось: внутри стоят функции, значения которых всегда положительны.

Следствия из области определения функции

Иногда для решения задачи B15 недостаточно просто найти вершину параболы. Искомое значение может лежать на конце отрезка , а вовсе не в точке экстремума. Если в задаче вообще не указан отрезок, смотрим на область допустимых значений исходной функции. А именно:

Обратите внимание еще раз: ноль вполне может быть под корнем, но в логарифме или знаменателе дроби — никогда. Посмотрим, как это работает на конкретных примерах:

Задача. Найдите наибольшее значение функции:

Под корнем снова квадратичная функция: y = 3 − 2x − x 2 . Ее график — парабола, но ветви вниз, поскольку a = −1

Выписываем область допустимых значений (ОДЗ):

3 − 2x − x 2 ≥ 0 ⇒ x 2 + 2x − 3 ≤ 0 ⇒ (x + 3)(x − 1) ≤ 0 ⇒ x ∈ [−3; 1]

Теперь найдем вершину параболы:

x 0 = −b /(2a ) = −(−2)/(2 · (−1)) = 2/(−2) = −1

Точка x 0 = −1 принадлежит отрезку ОДЗ — и это хорошо. Теперь считаем значение функции в точке x 0 , а также на концах ОДЗ:

y (−3) = y (1) = 0

Итак, получили числа 2 и 0. Нас просят найти наибольшее — это число 2.

Задача. Найдите наименьшее значение функции:

y = log 0,5 (6x − x 2 − 5)

Внутри логарифма стоит квадратичная функция y = 6x − x 2 − 5. Это парабола ветвями вниз, но в логарифме не может быть отрицательных чисел, поэтому выписываем ОДЗ:

6x − x 2 − 5 > 0 ⇒ x 2 − 6x + 5

Обратите внимание: неравенство строгое, поэтому концы не принадлежат ОДЗ. Этим логарифм отличается от корня, где концы отрезка нас вполне устраивают.

Ищем вершину параболы:

x 0 = −b /(2a ) = −6/(2 · (−1)) = −6/(−2) = 3

Вершина параболы подходит по ОДЗ: x 0 = 3 ∈ (1; 5). Но поскольку концы отрезка нас не интересуют, считаем значение функции только в точке x 0:

y min = y (3) = log 0,5 (6 · 3 − 3 2 − 5) = log 0,5 (18 − 9 − 5) = log 0,5 4 = −2

\(\blacktriangleright\) Для того, чтобы найти наибольшее/наименьшее значение функции на отрезке \(\) , необходимо схематично изобразить график функции на этом отрезке.
В задачах из данной подтемы это можно сделать с помощью производной: найти промежутки возрастания (\(f»>0\) ) и убывания (\(f»

\(\blacktriangleright\) Не стоит забывать, что наибольшее/наименьшее значение функция может принимать не только во внутренних точках отрезка \(\) , а также на его концах.

\(\blacktriangleright\) Наибольшее/наименьшее значение функции — это значение координаты \(y=f(x)\) .2 + 1}} = 0\qquad\Leftrightarrow\qquad x = 0\,.\] Производная существует при любом \(x\) .

2) Найдём промежутки знакопостоянства \(y»\) :


3) Найдём промежутки знакопостоянства \(y»\) на рассматриваемом отрезке \([-1; 1]\) :


4) Эскиз графика на отрезке \([-1; 1]\) :


Таким образом, наибольшего на \([-1; 1]\) значения функция достигает в \(x = -1\) или в \(x = 1\) . Сравним значения функции в этих точках.

\ Итого: \(2\) – наибольшее значение функции \(y\) на \([-1; 1]\) .

Ответ: 2

Задание 3 #2356

Уровень задания: Равен ЕГЭ

Найдите наименьшее значение функции \(y = \cos 2x\) на отрезке \(\) .

ОДЗ: \(x\) – произвольный.

1) \

Найдём критические точки (то есть внутренние точки области определения функции, в которых её производная равна \(0\) или не существует): \[-2\cdot \sin 2x = 0\qquad\Leftrightarrow\qquad 2x = \pi n, n\in\mathbb{Z}\qquad\Leftrightarrow\qquad x = \dfrac{\pi n}{2}, n\in\mathbb{Z}\,.2 — 12x + 36 + 2\ln 2)}\) .

С практической точки зрения наибольший интерес представляет использование производной для нахождения наибольшего и наименьшего значения функции. С чем это связано? Максимизация прибыли, минимизация издержек, определение оптимальной загрузки оборудования… Другими словами, во многих сферах жизни приходится решать задачи оптимизации каких-либо параметров. А это и есть задачи на нахождение наибольшего и наименьшего значения функции.

Следует отметить, что наибольшее и наименьшее значение функции обычно ищется на некотором интервале X , который является или всей областью определения функции или частью области определения. Сам интервал X может быть отрезком , открытым интервалом , бесконечным промежутком .

В этой статье мы будем говорить о нахождении наибольшего и наименьшего значений явно заданной функции одной переменной y=f(x) .

Навигация по странице.

Наибольшее и наименьшее значение функции — определения, иллюстрации.

Кратко остановимся на основных определениях.

Наибольшим значением функции , что для любого справедливо неравенство .

Наименьшим значением функции y=f(x) на промежутке X называют такое значение , что для любого справедливо неравенство .

Эти определения интуитивно понятны: наибольшее (наименьшее) значение функции – это самое большое (маленькое) принимаемое значение на рассматриваемом интервале при абсциссе .

Стационарные точки – это значения аргумента, при которых производная функции обращается в ноль.

Для чего нам стационарные точки при нахождении наибольшего и наименьшего значений? Ответ на этот вопрос дает теорема Ферма. Из этой теоремы следует, что если дифференцируемая функция имеет экстремум (локальный минимум или локальный максимум) в некоторой точке, то эта точка является стационарной. Таким образом, функция часто принимает свое наибольшее (наименьшее) значение на промежутке X в одной из стационарных точек из этого промежутка.

Также часто наибольшее и наименьшее значение функция может принимать в точках, в которых не существует первая производная этой функции, а сама функция определена.

Сразу ответим на один из самых распространенных вопросов по этой теме:»Всегда ли можно определить наибольшее (наименьшее) значение функции»? Нет, не всегда. Иногда границы промежутка X совпадают с границами области определения функции или интервал X бесконечен. А некоторые функции на бесконечности и на границах области определения могут принимать как бесконечно большие так и бесконечно малые значения. В этих случаях ничего нельзя сказать о наибольшем и наименьшем значении функции.

Для наглядности дадим графическую иллюстрацию. Посмотрите на рисунки – и многое прояснится.

На отрезке


На первом рисунке функция принимает наибольшее (max y ) и наименьшее (min y ) значения в стационарных точках, находящихся внутри отрезка [-6;6] .

Рассмотрим случай, изображенный на втором рисунке. Изменим отрезок на . В этом примере наименьшее значение функции достигается в стационарной точке, а наибольшее — в точке с абсциссой, соответствующей правой границе интервала.

На рисунке №3 граничные точки отрезка [-3;2] являются абсциссами точек, соответствующих наибольшему и наименьшему значению функции.

На открытом интервале


На четвертом рисунке функция принимает наибольшее (max y ) и наименьшее (min y ) значения в стационарных точках, находящихся внутри открытого интервала (-6;6) .

На интервале , о наибольшем значении никаких выводов сделать нельзя.

На бесконечности


В примере, представленном на седьмом рисунке, функция принимает наибольшее значение (max y ) в стационарной точке с абсциссой x=1 , а наименьшее значение (min y ) достигается на правой границе интервала. На минус бесконечности значения функции асимптотически приближаются к y=3 .

На интервале функция не достигает ни наименьшего, ни наибольшего значения. При стремлении к x=2 справа значения функции стремятся к минус бесконечности (прямая x=2 является вертикальной асимптотой), а при стремлении абсциссы к плюс бесконечности, значения функции асимптотически приближаются к y=3 . Графическая иллюстрация этого примера приведена на рисунке №8.

Алгоритм нахождения наибольшего и наименьшего значения непрерывной функции на отрезке .

Запишем алгоритм, позволяющий находить наибольшее и наименьшее значение функции на отрезке.

  1. Находим область определения функции и проверяем, содержится ли в ней весь отрезок .
  2. Находим все точки, в которых не существует первая производная и которые содержатся в отрезке (обычно такие точки встечаются у функций с аргументом под знаком модуля и у степенных функций с дробно-рациональным показателем). Если таких точек нет, то переходим к следующему пункту.
  3. Определяем все стационарные точки, попадающие в отрезок . Для этого, приравниваем ее к нулю, решаем полученное уравнение и выбираем подходящие корни. Если стационарных точек нет или ни одна из них не попадает в отрезок, то переходим к следующему пункту.
  4. Вычисляем значения функции в отобранных стационарных точках (если такие имеются), в точках, в которых не существует первая производная (если такие имеются), а также при x=a и x=b .
  5. Из полученных значений функции выбираем наибольшее и наименьшее — они и будут искомыми наибольшим и наименьшим значениями функции соответственно.

Разберем алгоритм при решении примера на нахождение наибольшего и наименьшего значения функции на отрезке.

Пример.

Найти наибольшее и наименьшее значение функции

  • на отрезке ;
  • на отрезке [-4;-1] .

Решение.

Областью определения функции является все множество действительных чисел, за исключением нуля, то есть . Оба отрезка попадают в область определения.

Находим производную функции по :

Очевидно, производная функции существует во всех точках отрезков и [-4;-1] .

Стационарные точки определим из уравнения . Единственным действительным корнем является x=2 . Эта стационарная точка попадает в первый отрезок .

Для первого случая вычисляем значения функции на концах отрезка и в стационарной точке, то есть при x=1 , x=2 и x=4 :

Следовательно, наибольшее значение функции достигается при x=1 , а наименьшее значение – при x=2 .

Для второго случая вычисляем значения функции лишь на концах отрезка [-4;-1] (так как он не содержит ни одной стационарной точки):

Решение.

Начнем с области определения функции. Квадратный трехчлен в знаменателе дроби не должен обращаться в ноль:

Легко проверить, что все интервалы из условия задачи принадлежат области определения функции.

Продифференцируем функцию:

Очевидно, производная существует на всей области определения функции.

Найдем стационарные точки. Производная обращается в ноль при . Эта стационарная точка попадает в интервалы (-3;1] и (-3;2) .

А теперь можно сопоставить полученные в каждом пункте результаты с графиком функции. Синими пунктирными линиями обозначены асимптоты.

На этом можно закончить с нахождением наибольшего и наименьшего значения функции. Алгоритмы, разобранные в этой статье, позволяют получить результаты при минимуме действий. Однако бывает полезно сначала определить промежутки возрастания и убывания функции и только после этого делать выводы о наибольшем и наименьшем значении функции на каком-либо интервале. Это дает более ясную картину и строгое обоснование результатов.

2 + Ьх = (ах + Ь) х. \ end {align} Следовательно, если $ (x, c) $ находится на кривой, то либо $ ax + b = 0 $, либо $ x = 0 $. Установив $ x_1 = — \ dfrac ba $ и $ x_2 = 0 $, мы можем подставить эти два значения за $ x $ и убедитесь, что действительно две точки $ \ left (- \ frac ba, c \ right) $ и $ (0, c) $ находятся на кривой.

Используя предположение, что кривая симметрична относительно вертикальной оси, вертикальная ось должна быть посередине между $ \ left (- \ frac ba, c \ right) $ и $ (0, c) $, то есть это линия $ x = — \ dfrac b {2a} $.2} {4a} $ только что «появился» в исходное уравнение в результате прямой подстановки.

Я думаю, что это может быть примерно так же, как «завершение квадрата» как чисто алгебраический метод.

5.1 Максимум и минимум

Точка локального максимума на функции является точка $ (x, y) $ на графике функции, координата $ y $ которой равна больше, чем все остальные координаты $ y $ на графике в точках «близких» к » $ (x, y) $. Точнее, $ (x, f (x)) $ является локальным максимумом, если существует — интервал $ (a, b) $ с $ локальной точкой минимума если он имеет локально наименьшую координату $ y $.Опять таки точнее: $ (x, f (x)) $ является локальным минимумом, если существует — интервал $ (a, b) $ с $ a локальным экстремумом является либо локальным минимумом, либо локальным максимумом.

Точки локального максимума и минимума весьма различимы на графике функции, и поэтому полезны для понимания формы график. Во многих прикладных задачах мы хотим найти самые большие или наименьшее значение, достигаемое функцией (например, мы можем захотеть найти минимальную стоимость, при которой может быть выполнено какое-либо задание) и так определение максимальных и минимальных точек будет полезно для применения проблемы тоже.Некоторые примеры локальных точек максимума и минимума показаны на рисунке 5.1.1.

Рисунок 5.1.1. Некоторые точки локального максимума ($ A $) и точки минимума ($ B $).

Если $ (x, f (x)) $ — это точка, в которой $ f (x) $ достигает локального максимума или минимума, и если производная $ f $ существует в $ x $, то граф имеет касательная линия и касательная линия должны быть горизонтальными. Это достаточно важно, чтобы сформулировать его как теорему, хотя мы не будем его доказывать.

Теорема 5.1.1 (теорема Ферма) Если $ f (x) $ имеет локальный экстремум при $ x = a $ и $ f $ дифференцируема в $ a $, тогда $ f ‘(a) = 0 $.$ \ qed $

Таким образом, единственный точки, в которых функция может иметь локальный максимум или минимум, являются точки, в которых производная равна нулю, как на левом графике в рисунок 5.1.1, или производная не определена, как на правом графике. Любое значение $ x $, для которого $ f ‘(x) $ равно нулю или не определено, называется критическое значение для $ f $, и точка $ (x, f (x)) $ на кривой называется критическая точка за $ f $. При поиске локальных точек максимума и минимума вы, скорее всего, сделать два вида ошибок: вы можете забыть, что максимум или минимум может возникнуть там, где производная не существует, поэтому забудьте проверить существует ли производная везде.2 $, и $ f ‘(0) = 0 $, но нет ни максимума, ни минимум на уровне $ (0,0) $.

Рисунок 5.1.2. Нет ни максимума, ни минимума, даже если производная равна нулю.

Поскольку производная равна нулю или не определена как в локальном максимуме, так и в точки локального минимума, нам нужен способ определить, какие из них действительно происходит. Большинство элементарный подход, но тот, который часто бывает утомительным или трудным, заключается в непосредственно проверить, находится ли $ y $ «рядом» с потенциалом максимум или минимум выше или ниже координаты $ y $ в точке представляет интерес.Конечно, «рядом» слишком много точек. чтобы проверить, но небольшая мысль показывает, что нам нужно проверить только два, при условии, что мы знать, что $ f $ непрерывно (напомним, это означает, что график $ f $ не имеет скачков и разрывов).

Предположим, например, что мы определили три точки, в которых $ f ‘$ равно нулю или не существует: $ \ ds (x_1, y_1) $, $ \ ds (x_2, y_2) $, $ \ ds (x_3, y_3) $, и $ \ ds x_15.1.3). Предположим, что мы вычисляем значение $ f (a) $ для $ \ ds x_1f (x_2) $? Нет: если бы были, график пошел бы вверх от $ (a, f (a)) $ в $ (b, f (b)) $, затем вниз до $ \ ds (x_2, f (x_2)) $ и где-то в между ними будет локальная точка максимума.(Это не очевидно; это результат теоремы о крайнем значении, теорема 6.1.2.) Но на этом локальном максимуме точка, производная от $ f $ была бы равна нулю или не существовала бы, но мы уже известно, что производная равна нулю или не существует только в $ \ ds x_1 $, $ \ ds x_2 $ и $ \ ds x_3 $. В результате одно вычисление говорит нам, что $ \ ds (x_2, f (x_2)) $ имеет наибольшую координату $ y $ любой точки на график около $ \ ds x_2 $ и слева от $ \ ds x_2 $. Мы можем сделать то же самое тест справа. Если мы обнаружим, что по обе стороны от $ \ ds x_2 $ значения меньше, то должен быть локальный максимум в $ \ ds (x_2, f (x_2)) $; если мы обнаруживаем, что по обе стороны от $ \ ds x_2 $ значения больше, то есть должен быть локальным минимумом в $ \ ds (x_2, f (x_2)) $; если мы найдем по одному из каждого, тогда нет ни локального максимума, ни минимума в $ \ ds x_2 $.

Рисунок 5.1.3. Тестирование на максимум или минимум.

Не всегда легко вычислить значение функции в конкретный момент. Задача облегчается наличием калькуляторы и компьютеры, но у них есть свои недостатки — они не всегда позволяют различать очень близкие значения вместе. Тем не менее, поскольку этот метод концептуально прост и иногда легко выполнить, вы всегда должны это учитывать.

Пример 5.2-1 $. Это определяется всюду и равен нулю в $ \ ds x = \ pm \ sqrt {3} / 3 $. Глядя сначала на $ \ ds x = \ sqrt {3} / 3 $, мы видим, что $ \ ds f (\ sqrt {3} / 3) = — 2 \ sqrt {3} / 9 $. Сейчас мы тестируем две точки по обе стороны от $ \ ds x = \ sqrt {3} / 3 $, убедившись, что ни один из них не находится дальше, чем ближайшее критическое значение; поскольку $ \ ds \ sqrt {3} -2 \ sqrt {3} / 9 $ и $ \ ds f (1) = 0> -2 \ sqrt {3} / 9 $, должен быть локальный минимум в $ \ ds x = \ sqrt {3} / 3 $. Для $ \ ds x = — \ sqrt {3} / 3 $ мы видим, что $ \ ds f (- \ sqrt {3} / 3) = 2 \ sqrt {3} / 9 $. На этот раз мы можем использовать $ x = 0 $ и $ x = -1 $, и находим, что $ \ ds f (-1) = f (0) = 0

Конечно, этот пример очень упрощен благодаря нашему выбору точек для test, а именно $ x = -1 $, $ 0 $, $ 1 $.Мы могли бы использовать другие значения, например $ -5 / 4 $, $ 1/3 $ и $ 3/4 $, но это сделало бы вычисления значительно более утомительный.

Пример 5.1.3 Найдите все локальные точки максимума и минимума для $ е (х) = \ грех х + \ соз х $. Производная равна $ f ‘(x) = \ cos x- \ sin x $. Это всегда определен и равен нулю всякий раз, когда $ \ cos x = \ sin x $. Напоминая, что $ \ cos x $ и $ \ sin x $ — координаты $ x $ и $ y $ точек на единичный круг, мы видим, что $ \ cos x = \ sin x $, когда $ x $ равен $ \ pi / 4 $, $ \ pi / 4 \ pm \ pi $, $ \ pi / 4 \ pm2 \ pi $, $ \ pi / 4 \ pm3 \ pi $ и т. д.Поскольку оба синуса и косинус имеют период $ 2 \ pi $, нам нужно только определить статус из $ x = \ pi / 4 $ и $ x = 5 \ pi / 4 $. Мы можем использовать $ 0 $ и $ \ pi / 2 $ для проверки критическое значение $ x = \ pi / 4 $. Получаем, что $ \ ds f (\ pi / 4) = \ sqrt {2} $, $ \ ds f (0) = 1

Мы используем $ \ pi $ и $ 2 \ pi $ для проверки критического значения $ x = 5 \ pi / 4 $. В соответствующие значения: $ \ ds f (5 \ pi / 4) = — \ sqrt2 $, $ \ ds f (\ pi) = — 1> — \ sqrt2 $, $ \ ds f (2 \ pi) = 1> — \ sqrt2 $, поэтому существует локальный минимум в $ x = 5 \ pi / 4 $, $ 5 \ pi / 4 \ pm2 \ pi $, $ 5 \ pi / 4 \ pm4 \ pi $ и т. Д. Более лаконично, есть локальные минимумы в $ 5 \ pi / 4 \ pm 2k \ pi $ для каждое целое число $ k $.2 & $ x \ neq 0 $ \ cr} $ (отвечать)

Пример 5.1.13 Для любого действительного числа $ x $ существует уникальное целое число $ n $ такое, что $ n \ leq x

Пример 5.1.14 Объясните, почему функция $ f (x) = 1 / x $ не имеет локального максимумы или минимумы.

Пример 5.1.15 Сколько критических точек может иметь квадратичная полиномиальная функция? (отвечать)

Пример 5.1.16 Докажите, что кубический многочлен может иметь не более двух критических точки. Приведите примеры, показывающие, что кубический многочлен может иметь нуль, одна или две критические точки.3 + cx + 1 $, где $ c $ является константой. Сколько и каких видов локальных крайностей существует? Ваш ответ должен зависеть от значения $ c $, то есть разного значения $ c $ дадут разные ответы.

Нахождение максимумов и минимумов с помощью производных

Где функция в верхней или нижней точке? Исчисление может помочь!

Максимум — это высокая точка, а минимум — это низкая точка:

В плавно меняющейся функции максимум или минимум всегда там, где функция выравнивает (за исключением седловой точки ).

Где он расплющивается? Где наклон равен нулю .

Где нулевой наклон? Производная говорит нам!

Давайте сразу рассмотрим пример:

Пример: в воздух подброшен мяч. Его высота в любой момент времени t определяется как:

ч = 3 + 14т — 5т 2

Какая максимальная высота?

Используя производные, мы можем найти наклон этой функции:

d dt h = 0 + 14-5 (2т)
= 14-10т

(См. Ниже этот пример, чтобы узнать, как мы нашли эту производную.)

Теперь найдите, когда наклон равен нулю :

14 — 10 т = 0

10 т = 14

т = 14/10 = 1,4

Наклон равен нулю при t = 1,4 секунды

А высота на тот момент:

h = 3 + 14 × 1,4 — 5 × 1,4 2

ч = 3 + 19,6 — 9,8 = 12,8

А так:

Максимальная высота 12,8 м (при t = 1.4 с)

Краткое руководство по производным инструментам

Производная определяет наклон функции.

В предыдущем примере мы взяли это:

ч = 3 + 14т — 5т 2

и придумал эту производную:

d dt h = 0 + 14-5 (2т)
= 14-10т

Что сообщает нам наклон функции в любое время t

Мы использовали эти производные правила:

  • Наклон постоянной значения (например, 3) составляет 0
  • Наклон линии , такой как 2x, равен 2, поэтому 14t имеет наклон 14
  • Квадрат Функция, такая как t 2 , имеет наклон 2t, поэтому 5t 2 имеет наклон 5 (2t)
  • И затем мы сложили их: 0 + 14 — 5 (2t)

Как узнать, что это максимум (или минимум)?

Мы видели это на графике! Но иначе… деривативы снова приходят на помощь.

Возьмем производную от наклона (вторая производная исходной функции):

Производная от 14 до 10t равна −10

Это означает, что наклон постоянно уменьшается (-10): при движении слева направо наклон начинается положительным (функция возрастает), проходит через ноль (плоская точка), а затем наклон становится отрицательным (функция падает). :


Уклон, который становится меньше (и идет через 0), означает максимум.

Это называется тестом второй производной

На графике выше я показал наклон до и после, но на практике мы проводим тест в точке, где наклон равен нулю :

Тест второй производной

Когда наклон функции равен нулю при x , а вторая производная при x равна:

  • меньше 0, это локальный максимум
  • больше 0, это локальный минимум
  • равно 0, тогда тест не пройден (хотя могут быть и другие способы выяснить)

«Вторая производная: меньше 0 — максимум, больше 0 — минимум»

Пример: Найдите максимумы и минимумы для:

y = 5x 3 + 2x 2 — 3x

Производная (наклон):

d dx y = 15x 2 + 4x — 3

Квадратичная с нулями в:

Могут ли они быть максимальными или минимальными? (Не смотрите пока на график!)

Вторая производная: y » = 30x + 4

При x = −3/5:

у » = 30 (−3/5) + 4 = −14

меньше 0, поэтому −3/5 — это локальный максимум

При x = +1/3:

у » = 30 (+1/3) + 4 = +14

больше 0, поэтому +1/3 — это локальный минимум

(Теперь вы можете посмотреть график.)

Слова

Верхняя точка называется максимумом (множественное число максимума ).

Нижняя точка называется минимумом (множественное число минимумов ).

Общее слово для максимума или минимума — экстремум (множественное число экстремум ).

Мы говорим локальный максимум (или минимум), когда могут быть более высокие (или более низкие) точки в другом месте, но не поблизости.

Еще один пример

Пример: Найдите максимумы и минимумы для:

y = x 3 — 6x 2 + 12x — 5

Производная:

d dx y = 3x 2 — 12x + 12

, который является квадратичным только с одним нулем при x = 2

Это максимум или минимум?

Вторая производная: y » = 6x — 12

При x = 2:

у » = 6 (2) — 12 = 0

это 0, поэтому тест не пройден

И вот почему:

Это точка перегиба («седловая точка»)… наклон действительно становится нулевым, но он не является ни максимальным, ни минимальным.

Должен быть дифференцируемым

И еще важный технический момент:

Функция должна быть дифференцируемой (производная должна существовать в каждой точке своего домена).

Пример: Как насчет функции f (x) = | x | (абсолютная величина) ?

| x | выглядит так:

При x = 0 он имеет очень резкое изменение!

Фактически, он не дифференцируется там (как показано на дифференцируемой странице).

Итак, мы не можем использовать производный метод для функции абсолютного значения.

Функция также должна быть непрерывной, но любая дифференцируемая функция также является непрерывной, поэтому мы покрыты.

Бизнес-расчет

В теории и приложениях мы часто хотим максимизировать или минимизировать какое-то количество. Инженер может захотеть максимизировать скорость нового компьютера или минимизировать тепловыделение, производимое устройством.Производитель может захотеть максимизировать прибыль и долю рынка или минимизировать отходы. Учащийся может захотеть максимизировать оценку по математике или свести к минимуму часы обучения, необходимые для получения определенной оценки.

Без исчисления мы знаем только, как найти оптимальные точки на нескольких конкретных примерах (например, мы знаем, как найти вершину параболы). Но что, если нам нужно оптимизировать незнакомую функцию?

Лучший способ без исчисления — это изучить график функции, возможно, используя технологию.Но наш вид зависит от выбранного окна просмотра — мы можем упустить что-то важное. Кроме того, таким образом мы, вероятно, получим только приблизительное значение. (В некоторых случаях этого будет достаточно.)

Calculus предоставляет способы резко сократить количество точек, которые нам нужно исследовать, чтобы найти точное местоположение максимумов и минимумов, и в то же время гарантировать, что мы не пропустили ничего важного.

Для просмотра этого видео включите JavaScript и рассмотрите возможность обновления до веб-браузера, который поддерживает видео HTML5

Локальные максимумы и минимумы

Прежде чем мы исследуем, как исчисление может помочь нам найти максимумы и минимумы, нам нужно определить концепции, которые мы будем развивать и использовать.

Определения (локальные максимумы и минимумы)

\ (f (x) \) имеет локальный максимум в \ (x = a \), если \ (f (a) \ geq f (x) \) для всех \ (x \) рядом с \ (a \ ).

\ (f (x) \) имеет локальный минимум в \ (x = a \), если \ (f (a) \ leq f (x) \) для всех \ (x \) вблизи \ (a \ ).

\ (f (x) \) имеет локальный экстремум в точке \ (x = a \), если \ (f (a) \) является локальным максимумом или минимумом .

Множественное число из них — максимумы и минимумы.Мы часто просто говорим «макс» или «мин»; это экономит много слогов.

В некоторых книгах написано «родственник» вместо «местный».

Процесс поиска максимумов или минимумов называется оптимизацией .

Точка — это локальный максимум (или минимум), если он выше (ниже) всех ближайших точек . Эти точки исходят из формы графика.

Определения (глобальные максимумы и минимумы)

\ (f (x) \) имеет глобальный максимум в \ (x = a \), если \ (f (a) \ geq f (x) \) для всех \ (x \) в области \ (е (х) \).

\ (f (x) \) имеет глобальный минимум в \ (x = a \), если \ (f (a) \ leq f (x) \) для всех \ (x \) в области \ (е (х) \).

\ (f (x) \) имеет глобальный экстремум в \ (x = a \), если \ (f (a) \) является глобальным максимумом или минимумом .

В некоторых книгах говорится «абсолютный» вместо «глобальный».

Точка — это глобальный максимум (или минимум), если он выше (ниже) каждой точки на графике. Эти точки исходят из формы графика и окна , через которое мы просматриваем график.

Обозначены локальные и глобальные крайние значения функции на рисунке 1. Вы должны заметить, что каждая глобальная крайность также является локальной, но есть локальные крайности, которые не являются глобальными крайностями.

Рисунок 1

Если \ (h (x) \) — высота земли над уровнем моря в точке \ (x \), то глобальный максимум \ (h \) равен \ (h \) (вершина горы. Эверест) = 29 028 футов. Локальный максимум \ (h \) для Соединенных Штатов равен \ (h \) (вершина горы Мак-Кинли) = 20 320 футов.Локальный минимум \ (h \) для Соединенных Штатов составляет \ (h \) (Долина Смерти) = -282 фута.

Пример 1

В таблице показано годовое количество зачисленных в крупный университет. В какие годы была максимальная или минимальная численность учащихся по математике? Каковы были максимальные и минимальные зачисления на математический факультет в мире?

Год 2000 2001 2002 2003 2004 2005 2006 2007 2008 2009 2010
Запись 1257 1324 1378 1336 1389 1450 1523 1582 1567 1545 1571

В 2002 и 2007 гг. Были локальные максимумы; в 2007 году мировой максимум составлял 1582 студента.В 2003 и 2009 гг. Были локальные минимумы; в 2000 г. глобальный минимум составлял 1257 студентов. Мы предпочитаем не думать о 2000 г. как о локальном минимуме или о 2010 г. как о локальном максимуме; однако некоторые книги будут включать конечные точки. Нам разрешено иметь глобальный максимум или глобальный минимум в конечной точке.

Для просмотра этого видео включите JavaScript и рассмотрите возможность обновления до веб-браузера, который поддерживает видео HTML5

Нахождение максимумов и минимумов функции

Как должна выглядеть касательная линия при локальном максимуме или минимуме? Посмотрите еще раз на эти два графика — вы увидите, что во всех крайних точках касательная линия горизонтальна (так что \ (f ‘= 0 \)).На синем графике есть один куспид — там касательная вертикальна (поэтому \ (f ‘\) не определено).

Это дает нам ключ к пониманию того, как находить экстремальные значения.

Определения

Критическое число для функции \ (f \) — это значение \ (x = a \) в области \ (f \), где либо \ (f ‘(a) = 0 \), либо \ (f ‘(a) \) не определено.

Критическая точка для функции f — это точка (a, f (a)), где a — критическое число f.2 — 4x + 3) = 3 (x — 1) (x — 3) \] Итак, \ (f ‘(x) = 0 \) при \ (x = 1 \) и \ (x = 3 \) (и никаких других значений \ (x \)). Нет мест, где \ (f ‘\) не определено.

Критические числа: \ (x = 1 \) и \ (x = 3 \). Итак, критические точки — это (1, 6) и (3, 2).

Это единственные возможные места локальных крайностей \ (f \). Мы еще не обсуждали, как определить, является ли какая-либо из этих точек на самом деле локальной крайностью \ (f \) или какой она может быть. Но мы можем быть уверены, что никакая другая точка не является локальной крайностью.3 \) не имеет локальных крайностей.

Помните этот пример! Недостаточно найти критические точки — мы можем только сказать, что \ (f \) может иметь локальный экстремум в критических точках .

Первые и вторые производные тесты

Эта критическая точка является максимумом или минимумом (или ни тем, ни другим)?

После того, как мы нашли критические точки \ (f \), у нас все еще есть проблема определения, являются ли эти точки максимумом, минимумом или ни одним из них.

Все графики на рисунке ниже имеют критическую точку в (2, 3). Из графиков видно, что точка (2,3) является локальным максимумом в (a) и (d), (2,3) является локальным минимумом в (b) и (e), и (2,3) ) не является локальным экстремумом в (c) и (f).

Критические числа дают только возможных точек крайностей, а некоторые критические числа не являются точками крайностей. Критические числа кандидатов для положений максимумов и минимумов.

\ (f ‘\) и экстремальные значения \ (f \)

Здесь показаны четыре возможных формы графиков — на каждом графике точка, отмеченная стрелкой, является критической точкой, где \ (f ‘(x) = 0 \). Что происходит с производной вблизи критической точки?

При локальном максимуме, например на графике слева, функция увеличивается слева от локального максимума, а затем уменьшается справа. Производная сначала положительна, затем отрицательна при локальном максимуме. При локальном min функция уменьшается влево и увеличивается вправо, поэтому производная сначала отрицательна, а затем положительна.Когда нет локального экстремума, функция продолжает увеличиваться (или уменьшаться) сразу после критической точки — производная не меняет знак.

Тест первой производной на экстремумы

Найдите критические точки f.

Для каждого критического числа c проверьте знак f ’слева и справа от c. Что происходит со знаком, когда вы двигаетесь слева направо?

  • Если \ (f ‘(x) \) изменяется с положительного на отрицательный при \ (x = c \), то \ (f \) имеет локальный максимум при \ ((c, f (c) ) \).
  • Если \ (f ‘(x) \) изменяется с отрицательного на положительный при \ (x = c \), то \ (f \) имеет локальный минимум при \ ((c, f (c)) \). 2 + 9x + 2 \) и классифицируйте их как локальный максимум, локальный минимум или ни то, ни другое.2 — 4x + 3) = 3 (x — 1) (x — 3) \). Здесь проще всего работать с факторизованной формой, так что давайте воспользуемся ею.

    В (1, 6) мы могли бы выбрать число немного меньше 1, чтобы подставить его в формулу для \ (f ‘\) — возможно, используйте \ (x = 0 \) или \ (x = 0.9 \). Тогда мы могли бы изучить его знак. Но нас не волнует числовое значение, все, что нас интересует, — это его знак. И для этого нам не нужно ничего подключать:

    • Если \ (x \) немного меньше 1, то \ (x-1 \) отрицательно, а \ (x-3 \) отрицательно.Итак, \ (f ‘= 3 (x — 1) (x — 3) \) будет pos (neg) (neg) = положительным.
    • Для \ (x \) немного больше 1, мы можем оценить \ (f ‘\) с числом больше 1 (но меньше 3, мы не хотим проходить следующую критическую точку!) — возможно \ (х = 2 \). Или мы можем сделать быстрый аргумент со знаком, как то, что мы сделали выше: для \ (x \) немного больше 1, \ (f ‘= 3 (x — 1) (x — 3) \) будет pos (pos) (neg) = отрицательный.

    Итак, \ (f ‘\) изменяется с положительного на отрицательный, что означает, что существует локальный максимум в (1, 6).

    В качестве другого подхода мы могли бы провести числовую линию и отметить критические числа:

    Мы уже знаем, что производная равна нулю или не определена в критических числах. На каждом интервале между этими значениями производная останется с тем же знаком. Чтобы определить знак, мы могли бы выбрать тестовое значение в каждом интервале и оценить производную в этих точках (или использовать знаковый подход, использованный выше).

    At (3, 2) \ (f ‘\) изменяется с отрицательного на положительный, поэтому существует локальный min в (3, 2).Это подтверждает то, что мы видели ранее на графике.

    Для просмотра этого видео включите JavaScript и рассмотрите возможность обновления до веб-браузера, который поддерживает видео HTML5

    Для просмотра этого видео включите JavaScript и рассмотрите возможность обновления до веб-браузера, который поддерживает видео HTML5

    \ (f » \) и экстремальные значения \ (f \)

    Вогнутость функции также может помочь нам определить, является ли критическая точка максимумом или минимумом или нет.Например, если точка находится внизу функции вогнутого вверх, то точка является минимумом.

    Тест второй производной на экстремумы

    Найдите все критические точки \ (f \). Для тех критических точек, где \ (f ‘(c) = 0 \), найдите \ (f’ ‘(c) \).

    • Если \ (f » (c) \ lt 0 \) (отрицательный), то \ (f \) вогнутая вниз и имеет локальный максимум в точке \ (x = c \).
    • Если \ (f » (c) \ gt 0 \) (положительный), то \ (f \) вогнутая вверх и имеет локальный минимум в точке \ (x = c \).2 — 30х + 24 \\ f » (x) = & 12x — 30 \ конец {выравнивание *} \]

      Затем нам просто нужно вычислить \ (f » \) для каждого критического числа:

      \ (x = 1 \): \ (f » (1) = 12 (1) -30 \ lt 0 \), поэтому существует локальный максимум в \ (x = 1 \).

      \ (x = 4 \): \ (f » (4) = 12 (4) -30 \ gt 0 \), поэтому существует локальный минимум в \ (x = 4 \).

      Для просмотра этого видео включите JavaScript и рассмотрите возможность обновления до веб-браузера, который поддерживает видео HTML5

      Многим студентам нравится Второй производный тест.Второй производный тест часто бывает проще использовать, чем первый производный тест. Вам нужно только найти знак одного числа для каждого критического числа, а не двух. И если ваша функция является полиномом, ее вторая производная, вероятно, будет более простой функцией, чем производная.

      Однако, если вам нужно правило продукта, правило частного или цепное правило, чтобы найти первую производную, поиск второй производной может потребовать много работы. Кроме того, даже если вторая производная проста, проверка второй производной не всегда дает ответ.Первый производный тест всегда даст вам ответ.

      Используйте любой тест, который хотите. Но помните — вы должны провести некоторый тест, чтобы убедиться, что ваша критическая точка на самом деле является локальным максимумом или минимумом.

      Для просмотра этого видео включите JavaScript и рассмотрите возможность обновления до веб-браузера, который поддерживает видео HTML5

      Для просмотра этого видео включите JavaScript и рассмотрите возможность обновления до веб-браузера, который поддерживает видео HTML5

      Глобальные максимумы и минимумы

      В приложениях мы часто хотим найти глобальную крайность; недостаточно знать, что критическая точка является локальной крайностью.

      Например, если мы хотим получить наибольшую прибыль, мы хотим получить наибольшую прибыль из всех. Как мы находим глобальные максимумы и минимумы?

      Есть еще несколько вещей, о которых стоит подумать.

      Крайние точки конечной точки

      Локальные экстремумы функции возникают в критических точках — это точки в функции, которые мы можем найти, подумав о форме (и используя производную, чтобы помочь нам). Но если мы смотрим на функцию на закрытом интервале, конечные точки могут быть крайними.Эти крайние значения конечных точек не связаны с формой функции; они связаны с интервалом, окном, через которое мы просматриваем функцию.

      На приведенном выше графике видно, что есть три критических точки — одна локальная минимальная, одна локальная максимальная и одна, которая не является ни одной из них. Но глобальный максимум, самая высокая точка из всех, находится в левой конечной точке. Глобальный минимум, самая низкая точка из всех, находится в правой конечной точке.

      Как определить, являются ли конечные точки максимальными или минимальными? Это проще, чем вы ожидали — просто подключите конечные точки вместе со всеми критическими числами и сравните \ (y \) — значения.2 — 6x — 9 = 3 (x + 1) (x — 3) \). Нам нужно найти критические точки, и нам нужно проверить конечные точки.

      \ (f ‘(x) = 3 (x + 1) (x — 3) = 0 \), когда \ (x = -1 \) и \ (x = 3 \). Конечными точками интервала являются \ (x = -2 \) и \ (x = 6 \).

      Теперь мы просто сравниваем значения \ (f \) при этих четырех значениях \ (x \):

      \ (х \) \ (е (х) \)
      -2 3
      -1 10
      3 -22
      6 59

      Глобальный минимум \ (f \) на \ ([-2, 6] \) равен -22, когда \ (x = 3 \), и глобальный максимум \ (f \) на \ ([ -2, 6] \) равно 59, когда \ (x = 6 \).

      Для просмотра этого видео включите JavaScript и рассмотрите возможность обновления до веб-браузера, который поддерживает видео HTML5

      Если есть только одна критическая точка

      Если функция имеет только одну критическую точку и это локальный максимум (или минимум), тогда это должен быть глобальный максимум (или минимум). Чтобы убедиться в этом, подумайте о геометрии. Посмотрите на график слева — есть локальный максимум, и график идет вниз по обе стороны от критической точки. Допустим, была какая-то другая точка, которая была выше — тогда график должен был бы развернуться.Но этот поворотный момент стал бы еще одним критическим моментом. Если есть только одна критическая точка, график никогда не развернется.

      Если сомневаетесь, нарисуйте это и посмотрите.

      Если вы пытаетесь найти глобальный максимум или минимум на открытом интервале (или на всей реальной линии), и имеется более одной критической точки, то вам нужно посмотреть на график, чтобы решить, существует ли глобальный максимум или мин. Убедитесь, что все ваши критические точки показаны на вашем графике, и что вы график за ними — это скажет вам то, что вы хотите знать.2 + 9х + 2 \).

      Ранее мы обнаружили, что (1, 6) — локальный максимум, а (3, 2) — локальный минимум. Это не замкнутый интервал, и есть две критические точки, поэтому мы должны обратиться к графику функции, чтобы найти глобальные max и min.

      График \ (f \) показывает, что точки слева от \ (x = 4 \) имеют \ (y \) — значения больше 6, поэтому (1, 6) не является глобальным максимумом. Аналогично, если \ (x \) отрицательно, \ (y \) меньше 2, поэтому (3, 2) не является глобальным min. Конечных точек нет, поэтому мы исчерпали все возможности.Эта функция не имеет глобального максимума или минимума.

      В поисках глобальных крайностей

      Единственные места, где функция может иметь глобальный экстремум, — это критические точки или конечные точки.

      • Если функция имеет только одну критическую точку, и это локальный экстремум, то это также глобальный экстремум.
      • Если есть конечные точки, найдите глобальные экстремумы, сравнивая \ (y \) — значения во всех критических точках и на конечных точках.
      • В случае сомнений нарисуйте функцию, чтобы быть уверенным.(Однако, если проблема явно не говорит вам об обратном, недостаточно, чтобы просто использовать график для получения ответа.)

      071Maxima.lbz

      071Maxima.lbz

      Теорема об экстремальных значениях

      Вот теорема о поведении непрерывных функций. Доказательство этой теоремы выходит за рамки этого курса, поэтому я просто сформулирую теорему, не доказывая ее.

      Теорема об экстремальном значении Пусть f (x) — непрерывная функция на отрезке [a, b] .Существует по крайней мере одна точка c в интервале [a, b] , в которой функция достигает своего максимального значения. То есть для всех x в интервале [a, b] мы имеем это. Точно так же существует по меньшей мере одна точка d , в которой f (x) достигает своего минимального значения. То есть для всех x в интервале [a, b] мы имеем это.

      Эта теорема — пример того, что математики называют теоремой существования .Теорема говорит нам, что максимальное и минимальное значения существуют, но не говорит нам, где и как их найти. Для этого нам потребуется разработать дополнительные инструменты.

      В поисках экстремальных значений

      Локальный и глобальный максимум

      Точка x = c называется локальным максимумом для функции f (x) , если f (c) больше или равно f (x) для всех значений x рядом с с . Чтобы сделать это определение более конкретным, мы должны сказать, что означает, что x находится рядом с c .

      Определение (локальный максимум) Мы говорим, что функция f (x) имеет локальный максимум при x = c , если существует интервал (a, b) , содержащий c , такой, что для всех х в (а, б) имеем.

      В качестве особого случая этого определения, если интервал (a, b) фактически равен c , мы также говорим, что c является глобальным максимумом .

      Имея это определение, мы готовы записать нашу первую полезную теорему.

      Теорема Если f (x) дифференцируема при x = c и принимает локальное максимальное значение при x = c , то

      Доказательство Предположим, что f (x) имеет максимальное значение при c . Посмотрите на его производную x = c :

      (1)

      Обратите внимание, что f (x) принимает максимальное значение при c . Для всех остальных х рядом,.Это говорит нам о том, что

      Отсюда мы видим, что числитель либо отрицателен, либо, возможно, 0. Чтобы вычислить предел (1), мы можем вычислить пределы с любого направления.

      (2)

      (3)

      Помните, f (x) c дифференцируемым при 90. Следовательно, предел (1) должен существовать. Единственный способ согласовать пределы (2) и (3) друг с другом — сказать, что предел должен быть равен 0.

      Комментарии к теореме о максимуме

      Теорема о максимальном значении очень часто используется не по назначению, поэтому стоит потратить немного времени на то, чтобы более внимательно изучить ее и понять, как ее правильно использовать. В сжатом виде в этой теореме говорится:

      Получаем контрапозитивную форму бесплатно:

      или

      Чаще всего проблемы вызывают то, что люди будут пытаться использовать обратную теорему:

      Последнее утверждение в целом неверно.

      В качестве простого контрпримера рассмотрим f (x) = x 3 при x = 0 . Хотя 0, когда x равно 0, f не имеет максимума при x = 0 .

      Несмотря на ограничения на ее использование, теорема по-прежнему очень полезна. Теорема говорит, что все точки максимума функции имеют производную, равную 0. Таким образом, если вы хотите найти, где находятся максимумы, проверьте места, где производная равна 0.Эти места называются критическими точками . Критическая точка не является автоматически точкой максимума, но поскольку все точки максимума являются критическими точками, критические точки — лучшее место для начала поиска максимальных точек.

      Аналогичная теорема верна для минимумов:

      Теорема Если f дифференцируем при x = c и принимает локальное минимальное значение при x = c (то есть для всех x около c ), то

      Ищем максимумы и минимумы

      Мы установили, что крайние точки являются критическими точками, поэтому мы знаем, что нужно искать критические точки.Проблема в том, что, обнаружив критическую точку, мы не знаем, что с ней делать. Есть две стратегии, которые мы можем использовать для решения этой проблемы.

      Проверка значения Сравните значение функции с ближайшими значениями. Если критической точкой является, скажем, максимум, значение функции в этой точке будет больше, чем значение в соседних точках.

      Тест первой производной Если критическая точка является максимальной, функция должна возрастать (положительная производная) слева от этой точки и уменьшаться (отрицательная производная) справа.

      Вот пример. Рассмотрим функцию

      Эта функция имеет производную

      , который равен 0 при x = 0 и x = 1 . Это критические точки. Чтобы увидеть, являются ли эти критические точки крайними точками, мы можем посмотреть на поведение производной вблизи этих критических точек. Вот что делает производная в нескольких точках около 0 и 1.

      Это показывает, что когда x приближается к 0, производная становится отрицательной, переходит в 0, а затем снова становится отрицательной.Это означает, что точка x = 0 не является ни максимальной, ни минимальной точкой. Проходя через 1, мы видим функцию, которая убывает, выравнивается, а затем снова увеличивается. Это ясно указывает на то, что функция имеет минимальное значение при x = 1 . Если мы построим график, то увидим, что это действительно так.

      Второй тест производной

      Вторая производная также предоставляет полезную информацию о форме для функций.Чтобы убедиться в этом, давайте посмотрим на пример функции, которая везде имеет положительную вторую производную.

      Как вы можете видеть на графике этой функции, она имеет форму чаши с чашей, установленной вертикально.

      Форма чаши здесь соответствует положительной второй производной. Форма называется вогнутой вверх (или мы говорим, что функция вогнута вверх). Простая модификация последнего примера даст нам пример функции, у которой вторая производная везде отрицательна.

      Эта форма, которая соответствует отрицательной второй производной, называется вогнутой вниз.

      Большинство функций чередуются между вогнутым вверх и вниз. Рассмотрим на примере

      Вторая производная положительна на части оси абсцисс. Эта часть предоставлена ​​

      6 x — 4> 0

      6 x> 4

      Аналогично, вторая производная отрицательна по остальной части оси абсцисс.

      6 x — 4 <0

      6 x <4

      Точка, в которой вторая производная изменяется с положительной на отрицательную, называется точкой перегиба. Чтобы найти его, ищите

      В этом примере единственная точка перегиба расположена в точке x = 2/3 . Если мы рассмотрим график этой функции, мы можем ясно увидеть вогнутые и вогнутые вниз части графика.

      Мы можем резюмировать то, что мы здесь узнали, сказав

      Теорема Предположим, что f (x) дифференцируема всюду на открытом интервале (a, b) .Если x = c является критической точкой функции f (x) , расположенной где-то в интервале (a, b) и, то c является локальным максимумом для f (x) . Аналогично, если, то c является локальным минимумом для f (x) .

      Пример

      Воспользуемся тем, что мы только что узнали о критических точках, чтобы построить график функции

      Критические точки расположены по адресу

      или

      Чтобы определить точную природу этих критических точек, мы можем использовать любой из трех вышеупомянутых методов.

      Первый способ — составить таблицу значений функции f (x) в этих точках и в точках рядом:

      756
      х
      -0,8 1,31
      1,32

      906

      1.01
      0 1
      0.1 1.01

      Эта информация показывает, что критическая точка — это локальный максимум, а критическая точка x = 0 — это локальное минимальное значение.

      Второй метод — изучить поведение первой производной вблизи критических точек.

      0 Третий метод — вычислить вторую производную в критических точках.

      Это показывает, что функция вогнута вверх в точке 0 (что создает там локальный минимум) и вогнута вниз в точке (что создает там локальный максимум).

      Сюжет все это подтверждает:

      x
      -0,8 0,448
      0
      -876 -0,552
      -0,1 -0,1995
      0 0 0

      Домашнее задание

      32, 33, 41, 42, 52, 53, 70, 73

      Модуль 13 — Экстремальные значения функций

      В этом уроке вы узнаете об абсолютных и локальных экстремальных точках и определите экстремальные точки из набора критических точек и конечных точек.


      Задачи оптимизации — одно из самых важных приложений дифференциального исчисления, потому что мы часто хотим знать, когда выход функции достигает максимума или минимума.В таких задачах может быть наибольшее или наименьшее выходное значение на всем интересующем входном интервале или в локальной окрестности входного значения. Как абсолютные, так и локальные максимальные и минимальные значения представляют интерес во многих контекстах.

      Абсолютные экстремальные значения функции

      Когда выходное значение функции является максимумом или минимумом во всем домене функции, значение называется абсолютным максимумом или абсолютным минимумом , как определено ниже.

      Пусть f будет функцией с доменом D и пусть c будет фиксированной константой в D . Тогда выходное значение f ( c ) является

      1. абсолютное максимальное значение из f на D тогда и только тогда, когда f ( x ) f ( c ) для всех x в D .
      2. абсолютное минимальное значение из f на D тогда и только тогда, когда f ( c ) f ( x ) для всех x дюймов D .

      Абсолютные экстремальные значения — пример

      Домен f ( x ) = x 2 — это все действительные числа, а диапазон — все неотрицательные действительные числа. График на рисунке ниже показывает, что функция не имеет абсолютного максимального значения и имеет абсолютный минимум 0, что происходит при x = 0.

      [-5, 5, 1] ​​x [-2, 10, 1]

      Абсолютные экстремальные значения в ограниченной области

      Если домен f ( x ) = x 2 ограничен [-2, 3], соответствующий диапазон будет [0, 9].Как показано ниже, график на интервале [-2, 3] предполагает, что f имеет абсолютный максимум 9 при x = 3 и абсолютный минимум 0 при x = 0.

      Два приведенных выше примера показывают, что существование абсолютных максимумов и минимумов зависит от области определения функции.

      Теорема об экстремальном значении

      Теорема 1 ниже называется теоремой об экстремальном значении.Он описывает условие, которое гарантирует, что функция имеет как абсолютный минимум, так и абсолютный максимум. Теорема важна, потому что она может направлять наши исследования при поиске абсолютных крайних значений функции.

      Теорема 1 Если f является непрерывным на отрезке [ a , b ], то f имеет как абсолютное максимальное значение, так и абсолютное минимальное значение на интервале.

      Эта теорема говорит, что непрерывная функция, которая определена на закрытом интервале , должна иметь как абсолютное максимальное значение, так и абсолютное минимальное значение. В нем не говорится о том, как найти крайние значения.

      Локальные экстремальные значения функции

      Одним из наиболее полезных результатов исчисления является то, что абсолютные экстремальные значения функции должны поступать из списка локальных экстремальных значений, и эти значения легко найти с помощью первой производной функции.

      Локальные экстремальные значения, как определено ниже, — это точки максимума и минимума (если они есть), когда область ограничена небольшой окрестностью входных значений.

      Пусть c будет внутренней точкой области определения функции f . Тогда функция f имеет

      1. локальный максимум при c тогда и только тогда, когда f ( x ) f ( c ) для всех x в некотором открытом интервале, содержащем c .
      2. местный минимум при c тогда и только тогда, когда f ( c ) f ( x ) для всех x в некотором открытом интервале, содержащем c .

      Конечные точки как локальные экстремумы

      Приведенное выше определение локальных экстремумов ограничивает входное значение внутренней точкой области.Определение можно расширить, включив в него конечные точки интервалов.

        Функция f имеет локальный максимум или локальный минимум в конечной точке c своего домена, если соответствующее неравенство выполняется для всех x в некотором полуоткрытом интервале, содержащемся в домене и имеющем c в качестве одной конечной точки. .

      Из определений ясно, что для областей, состоящих из одного или нескольких интервалов, любая абсолютная крайняя точка также должна быть локальной крайней точкой.Итак, абсолютные экстремумы можно найти, исследуя все локальные экстремумы.

      Кандидаты в местные очки экстремальной ценности

      Теорема 2 ниже, которую также называют теоремой Ферма, определяет кандидатов в локальные экстремальные точки.

      Теорема 2 Если функция имеет локальное максимальное значение или локальное минимальное значение во внутренней точке c ее области и если f ‘ существует в c , то f’ ( c ) = 0.

      Нахождение экстремальных значений функции

      Теорема 2 гласит, что если функция имеет первую производную во внутренней точке, где есть локальный экстремум, то производная должна быть равна нулю в этой точке. Он не говорит, что каждая точка, в которой первая производная равна нулю, должна быть локальным экстремумом. В силу теоремы 2 при нахождении экстремальных значений функции необходимо учитывать лишь несколько моментов.Эти точки состоят из точек внутренней области, где f ‘ ( x ) = 0, точек внутренней области, где f’ не существует, и конечных точек области, которые не покрываются теоремой.

      Критические точки

      Критическая точка — это внутренняя точка в области определения функции, в которой f ‘ ( x ) = 0 или f’ не существует.Таким образом, единственными возможными кандидатами на координату x экстремальной точки являются критические и конечные точки.

      Нахождение экстремальных значений с помощью методов исчисления

      Найдите локальные и абсолютные экстремальные значения f ( x ) = x 2 на замкнутом интервале [-2, 3] с помощью исчисления. Здесь применима теорема 1, поэтому мы точно знаем, что эта функция должна иметь абсолютные экстремумы в этой области.

      Обратите внимание на следующее:

      1. f ‘ ( x ) = 2 x , который равен нулю только при x = 0 и существует при всех значениях f в [-2, 3]. Следовательно, x = 0 — единственная критическая точка для f .
      2. Значения f в конечных точках равны f (-2) = 4 и f (3) = 9.

      Сравнивая выходные значения, когда x = -2, x = 0 и x = 3, можно определить абсолютные экстремумы.

      1. f имеет локальный минимум 0 при x = 0, что также является абсолютным минимумом.
      2. f имеет локальный максимум 4 при x = -2 и локальный максимум 9 при x = 3.Абсолютный максимум f равен 9.

      Просмотрите график функции в ограниченной области. График подтверждает приведенные выше результаты.

      [-2, 3, 1] x [-2, 10, 1]

      13.1.1 Найдите крайние значения f ( x ) = x 2 на [-4, 2], используя методы исчисления, а затем подтвердите свои ответы, нарисовав график.Нажмите здесь, чтобы получить ответ.

      Методы исчисления дают результаты, которые могут быть подтверждены графиками, а графики могут помочь в обнаружении экстремальных значений, как показано в следующем примере.

      Экстремальные значения f ( x ) = x 2/3 на [-2, 4]

      Найдите крайние значения f ( x ) = x 2/3 в ограниченной области [-2, 4], просмотрев график, а затем используя методы вычислений.(2/3) в Y 1 .

    • Отобразите график в окне [-2, 4,1] x [-1, 3,1].

    Функция имеет абсолютный минимум около x = 0 и два локальных максимума, которые возникают в конечных точках ограниченного домена. Абсолютный максимум достигается в правой конечной точке ограниченного домена.

    Теперь определите крайние точки, используя методы исчисления.

    • Используйте правило мощности, чтобы найти f ‘:

    Производная, , не равно 0 в любом месте [-2, 4], поэтому из этого условия не возникает критическая точка, но f ‘ не существует при x = 0, что означает, что x = 0 является критическим точка. Следовательно, единственная критическая точка f находится при x = 0.

    Используйте функцию Value экрана Graph для вычисления значений f в критической точке и на конечных точках ограниченного домена [-2, 4].

    • Из графика f press [CALC] и выберите 1: значение.
    • Вычислите f при x = -2, x = 0 и x = 4, введя -2, 0 и 4 соответственно.

    Крайние значения можно резюмировать следующим образом:

    1. f имеет локальный и абсолютный минимум 0 при 0.
    2. Значение f при x = -2 составляет приблизительно 1,587, а значение при x = 4 составляет приблизительно 2,520. Каждое из них является локальным максимальным значением.
    3. Абсолютное максимальное значение f составляет примерно 2.520 при x = 4.

    Экстремальные значения

    В предыдущих примерах мы имели дело с непрерывными функциями, определенными на отрезках. В таком случае теорема 1 гарантирует, что будет как абсолютный максимум, так и абсолютный минимум. В этом примере область не является закрытым интервалом, и теорема 1 не применяется. Крайние значения могут быть найдены с помощью процедуры, аналогичной описанной выше, но необходимо следить за тем, чтобы экстремумы действительно существовали.

    Обратите внимание, что домен f равен (-2, 2), потому что подкоренное выражение должно быть неотрицательным, а знаменатель должен быть ненулевым.

    • График в окне просмотра [-4, 4, 1] x [-2, 4, 1].

    График показывает, что существует абсолютный минимум около 0,5 при x = 0. Также есть локальные максимумы около 2.5, когда x = -2 и x = 2. Однако f не определен при x = -2 и x = 2, поэтому они не могут быть локальными максимумами.

    Методы исчисления требуют, чтобы конечные точки области и критические точки были идентифицированы. Домен f — это (-2, 2), открытый интервал, поэтому конечных точек нет. Критические точки определяются с помощью производной, которая находится с помощью правила цепочки.

    Производная равна 0 при x = 0 и не определена при x = -2 и x = 2.Поскольку -2 и 2 не находятся в области f , единственная критическая точка — x = 0.

    Поскольку x перемещается от 0 в любом направлении, знаменатель f ( x ) становится меньше, а f ( x ) становится больше. Таким образом, f имеет абсолютный минимум 0,5 при x = 0.

    Абсолютного максимума не существует. Это не нарушает теорему об экстремальных значениях, поскольку функция не определена на отрезке.Поскольку абсолютный максимум должен происходить в критической точке или конечной точке, а x = 0 является единственной такой точкой, абсолютного максимума быть не может.

    y = x 3 в окне [-3, 3 1] x [-2, 2, 1] в окне [-3, 3, 1] x [-2, 2, 1]

    Обратите внимание, что производная от y = x 3 равна y ‘ = 3 x 2 , а производная от y = x 1/3 равна .

    Первая производная от y = x 3 равна нулю, когда x = 0, а первая производная от y = x 1/3 не существует при x = 0. Хотя x = 0 является критической точкой для обеих функций, ни одна из них не имеет там экстремального значения.

    Помимо поиска критических точек с помощью методов исчисления, просмотр графика функции должен помочь определить экстремальные значения.

    В этих двух примерах обратите внимание, что первая производная положительна по обе стороны от x = 0. В уроке 13.2 мы будем использовать тест первой производной, где знак производной по обе стороны от критической точки используется для определения является ли критическая точка локальным максимумом, локальным минимумом или ни одним из них.

    Global Extrema

    Global Extrema

    Глобальные экстремумы


    На последней странице вы узнали, как находить локальные экстремумы; один часто больше интересует поиск глобальных экстремумов :

    Мы говорим, что функция f ( x ) имеет глобальный максимум при x = x 0 на интервале I , если для всех .Точно так же функция f ( x ) имеет глобальный минимум при x = x 0 на интервале I , если для всех .

    Если f ( x ) — непрерывная функция на замкнутом ограниченном интервале [ a , b ], тогда f ( x ) будет иметь глобальный максимум и глобальный минимум на [ a , b ]! (Хотя это непросто доказать).

    С другой стороны, если интервал не ограничен и не замкнут, то нет гарантии, что непрерывная функция f ( x ) будет иметь глобальные экстремумы.Примеры: f ( x ) = x 2 не имеет глобального максимум на интервале , функция не имеет глобального минимума на интервале (0,1).

    Как найти глобальные экстремумы? К сожалению, не все глобальные экстремум также является локальным экстремумом:

    Пример. Рассмотрим функцию f ( x ) = ( x -1) 2 , для . Единственная критическая точка — x = 1. И первый или второй производный тест будет означать, что x = 1 является локальным минимумом.Смотрящий на графике (см. ниже) мы видим, что правая конечная точка интервала [0,3] — глобальный максимум.

    Это побуждает нас представить новую концепцию конечной точки . Экстремумы . Действительно, если c является конечной точкой домена f ( x ), тогда f ( x ), как говорят, имеет максимум конечной точки на c iff для всех x в домене близком к c .Аналогично один может определить концепцию минимума конечной точки.

    Впрочем, новости не так уж и плохи. Если f ( x ) дифференцируется на интервал I , тогда:

    Каждый глобальный экстремум — это локальный экстремум или конечная точка. экстремум.

    Это предполагает следующую стратегию поиска глобальных экстремумов:

    • Найдите критические точки.
    • Укажите конечные точки рассматриваемого интервала.
    • Глобальные экстремумы f ( x ) могут возникать только в этих точках! Оцените f ( x ) в этих точках чтобы проверить, где находятся глобальные максимумы и минимумы.

    Пример. Найдем глобальные экстремумы функции f ( x ) = x e x на интервале [0.1,3.5]. Функция f ( x ) всюду дифференцируема, ее производная f ‘( x ) = e x xe x = (1- x ) e x только при х = 1.Таким образом x = 1 — единственная критическая точка. Добавьте конечные точки интервал x = 0,1 и x = 3,5, и оценить f ( x ):

    Таким образом, глобальный минимум происходит при x = 0,1, глобальный максимум происходит при x = 1.


    [Назад] [Следующий] [Тригонометрия] [Исчисление] [Геометрия] [Алгебра] [Дифференциальные уравнения] [Комплексные переменные] [Матричная алгебра] Домашняя страница S.

Author: alexxlab

Добавить комментарий

Ваш адрес email не будет опубликован. Обязательные поля помечены *